Tag Archives: PTNK

ĐỀ THI THỬ VÀO 10 PHỔ THÔNG NĂNG KHIẾU – TOÁN CHUNG

THỜI GIAN LÀM BÀI 120 PHÚT

PHẦN 1. TRẮC NGHIỆM (2 ĐIỂM)

Câu 1. Biểu thức $\sqrt{\frac{1}{1-2 x+x^2}}$ xác định khi và chỉ khi:
A. $x>1$
B. $x \geq 1$
C. $x \in R$
D. $x \neq 1$

Câu 2. Đường tròn tâm $O$ bán kính $R$ có $M A, M B$ là hai tiếp tuyến của $(\mathrm{O})(A, B$ là các tiếp điểm). Biết $\widehat{A O B}=90^{\circ}$, chu vi tam giác $M A B$ là:
A. $2 R$
B. $R \sqrt{2}+2$
C. $(2+\sqrt{2}) R$
D. $R \sqrt{2}$

Câu 3. Cho hai đường thẳng $\left(d_1\right): y=\left(2 m^2+3\right) x-3 m+1$ và $\left(d_2\right): y=5 x-2$. Hai đường thẳng trùng nhau khi:
A. $m=-1$
B. $m=1$
C. $m \neq 1$
D. $m \in{1 ;-1}$

Câu 4. Đường thẳng $\Delta: y=m x+n-2$ đi qua gốc tọa độ và điểm $A(-1 ; 3)$. Tính $m+2 n$.
A. 1
B. -2
C. -3
D. 2

Câu 5. Rút gọn biểu thức $T=\frac{\sqrt{x^4(x-y)^2}}{x^2-y^2}$ với $x<y<0$ bằng:
A. $\frac{x^2}{x-y}$
B. $\frac{-x^2}{x-y}$
C. $\frac{-x^2}{x+y}$
D. $\frac{x^2}{x+y}$

Câu 6. Câu nào sau đây đúng?
A. $|A|+|B|=0 \Leftrightarrow\left[\begin{array}{l}A=0 \\\ B=0\end{array}\right.$
C. $\sqrt{A}=|B| \Leftrightarrow\left\{\begin{array}{l}B \geq 0 \\\ A=B^2\end{array}\right.$
B. $(A-B)^2>0 \Leftrightarrow A \neq B$
D. $B, C$ đều đúng.

Câu 7. Cho đường tròn tâm $O$ có bán kính $2 R$ và một dây cung có độ dài bằng $2 R$. Khoảng cách từ tâm $O$ đến dây cung này là:
A. $R$
B. $\frac{R \sqrt{3}}{2}$
C. $R \sqrt{2}$
D. $R \sqrt{3}$

Câu 8. Gọi $\left(x_0, y_0\right)$ là nghiệm của hệ phương trình: $\left\{\begin{array}{l}2 x^2+y^2=5 \\\ x^2-y^2=1\end{array}\right.$. Tính $\frac{x_0}{y_0}$ biết $y_0<$ $0<x_0$.
A. -2
B. $\sqrt{2}$
C. $-\sqrt{2}$
D. 2

Câu 9. Tìm $m$ để parabol $(P): y=(m-2) x^2$ và đường thẳng $(D): y=2 x-3$ cắt nhau tại hai điểm phân biệt:
A. $m<\frac{7}{3}$ và $m \neq 2$

C. $m>\frac{7}{3}$ và $m \neq 2$
B. $m \geq \frac{7}{3}$ và $m \neq 2$
D. $m \leq \frac{7}{3}$ và $m \neq 2$

Câu 10. Cho tam giác $A B C$ có đường cao $A H$. Nếu $B C=2 A H$ và $\tan B=1$ thì tam giác $A B C$ là tam giác gì?
A. Tam giác nhọn
C. Tam giác vuông
B. Tam giác vuông cân
D. Tam giác cân

PHẦN TỰ LUẬN (8 ĐIỂM)

Bài 1. (1,5 điểm)
(a) Cho $M=\frac{3 \sqrt{x}-3}{4} \cdot\left(\frac{\sqrt{x}+1}{\sqrt{x}-1}-\frac{\sqrt{x}-1}{\sqrt{x}+1}\right) ; N=1-\frac{\sqrt{x}}{x-2}$ với $x \geq 0 ; x \neq$ $1 ; x \neq 2$.
Tìm $x$ biết $M \cdot N=6$.
(b) $\triangle A B C$ có $A D$ là đường phân giác của $\widehat{B A C}(D \in B C)$. Biết $A C=A B+B D$ và $\widehat{A B C}=60^{\circ}$. Lấy điểm $E$ trên đoạn thẳng $A C$ sao cho $A E=A B$. Đặt $\widehat{B A D}=x^{\circ}$ và $\widehat{A C B}=y^{\circ}$. Tìm $x, y$.

Bài 2. (2 diểm)
(a) Giải phương trình: $\left(-2 x^2+3 x+5\right) \cdot(\sqrt{1-2 x}-\sqrt{x+4}+1)=0$.
(b) Trong một ngày hội của trường, các lớp được yêu cầu tổ chức một gian hàng ẩm thực trong hai ngày. Lớp 10T dự định sẽ bán xiên thịt nướng, chi phí bỏ ra cho một xiên thịt nướng là 10000 đồng và số lượng xiên nướng chuẩn bị cho hai ngày là như nhau. Ngày thứ nhất, lớp bán hết số thịt đã chuẩn bị và lời 1000000 đồng. Sang ngày thứ hai, lớp tăng giá bán lên $20 \%$ và bán được $\frac{3}{4}$ số xiên thịt; với số xiên thịt còn lại lớp quyết định giảm về giá ban đầu, tuy nhiên khi còn 30 xiên thịt cuối lớp không bán mà để cho các bạn trong lớp tham gia bán hàng ăn. Biết số tiền lời ngày thứ hai bằng ngày thứ nhât, hỏi giá bán một xiên thịt ban đầu là bao nhiêu?

Bài 3. (1,5 điểm) Cho phương trình: $\frac{-3 x^2-2 m x+1-m}{x-1}=0$
(a) Phương trình (1) nhận $x=\frac{1}{3}$ là nghiệm. Tìm nghiệm còn lại của phương trình.
(b) Tìm $m$ để phương trình (1) có hai nghiệm phân biệt $x_1, x_2$ thỏa:
$$
3 x_1+6 x_2-3 x_1 x_2=m+2
$$
Bài 4. (3 diểm) Cho $\triangle A B C$ cân tại $A$ nội tiếp đường tròn tâm $O$ có $\widehat{B A C}=30^{\circ}$ và $B C=a$.
(a) Chứng minh tam giác $O B C$ đều, tính diện tích tam giác $O B C$.
(b) Gọi $M$ là trung điểm của $O B, C M$ cắt $(O)$ tại $K$ khác $C . O B$ cắt $A C$ tại $D$. Chứng minh tứ giác $O C B K$ là hình thoi và tính $\widehat{A D K}$.
(c) Trên đoạn $D C$ lấy điểm $E$ sao cho $A D=D E$. Chứng minh $A K \perp O E$ và $A C$ tiếp xúc với đường tròn ngoại tiếp tam giác $O E B$.

HẾT

Đề thi và đáp án chọn đội dự tuyển PTNK năm 2022

Thời gian làm bài 120 phút.

Bài 1. Cho $a, b, c \geq 0$ thỏa $a^2+b^2+c^2=1$. Tìm giá trị lớn nhất và giả trị nhỏ nhất của biểu thức $P=a b+b c+c a-2(a+b+c)$.

Bài 2. Cho $k, n \in Z^{+}$, có bao nhiêu đơn ánh từ $\{1, 2, \cdots, 2k+1\} \to \{1, 2, \cdots, 2n\}$ thỏa $f(1) < f(2) < \ldots < f(k) < f(k+1) > f(k+2)>\ldots> f(2 k)>f(2 k+1)$ và $f(k+1) \neq 2 n-2$.

Bài 3. Cho $n$ là số nguyên dương, kí hiệu $a(n)=1+2+\ldots+n$ và $b(n)=1^2+2^2+\ldots+n^2$. Hỏi có tồn tại số $n$ sao cho $2(n+1) a(n)+3 b(n)-1$ là số chính phương?

Bài 4. Cho tam giác $A B C$ có $2 A=5 B=10 C$. Phân giác trong $B D$ cẳt trung tuyển $C M$ tại I. Một đường thẳng $d$ đi qua $D$ vuông góc với $A C$ cắt $B C$ và $A I$ lần lượt tại $E$ và $K . A E$ cắt $C K$ tại $F$. Chứng minh: $M F$ song song $B K$.

Lời giải tham khảo

Bài 1. Đặt $t=a+b+c$ ta có $a(1-a) \geq 0, b(1-b) \geq 0, c(1-c) \geq$, suy ra $a+b+c \geq$ $a^2+b^2+c^2=1$, và $(a+b+c)^2 \leq 3\left(a^2+b^2+c^2\right)=3$, suy ra $a+b+c \leq \sqrt{3}$ Ta có $1=(a+b+c)^2-2(a b+b c+a c) \Rightarrow a b+b c+c a=\frac{t^2-1}{2}$.
Do đó $P=\frac{t^2-1}{2}-2 t=\frac{1}{2} t^2-2 t-\frac{1}{2}$ với $1 \leq t \leq \sqrt{3}$.
Khảo sát hàm bậc hai trong đoạn ta có $\max P=-2$ khi $t=1$ và $\min P=1-2 \sqrt{3}$.
Vậy $\max P=-2$ khi $a=1, b=c=0$ và min $P=1-2 \sqrt{3}$ khi $a=b=c=\frac{1}{\sqrt{3}}$.

Bài 2. Do đó $f$ là đơn ánh, $\operatorname{Im} f$ là một tập con có $2 k+1$ phần tử của $A$, mặt khác $f(k+1)$ là giá trị lớn nhất nên $\operatorname{Im} f$ có giá trị lớn nhất khác $2 n-2$.
Ta đếm số tập con có $2 k+1$ phần tử của $A$ mà phần tử lớn nhất khác $2 n-2$. Số tập con có $2 k+1$ của $A$ là $C_{2 n}^{2 k+1}$, số tập con có $2 k+1$ mà có phần tử lớn nhất $2 n-2$ là bằng với số tập con có $2 k$ phần tử của ${1,2, \cdots 2 k-3}$, là $C_{2 n-3}^{2 k}$.
Do đó theo nguyên lí bù trừ số tập con có $2 k+1$ của tập $A$ mà phần tử lớn nhất khác $2 n-2$ là $\left(C_{2 n}^{2 k+1}-C_{2 n-3}^{2 k}\right)$.
Tiếp theo ta đếm số đơn ánh từ ${1,2, \cdots, 2 k+1}$ tới $A^{\prime}=\left\{a_1, a_2, \cdots, a_{2 k+1}\right\}$ thỏa đề bài, ta có $f(k+1)=a_{2 k+1}$, nên số đơn ánh bằng số cách chọn $k$ phần tử từ $A^{\prime}$ nên bằng $C_{2 k}^k$.
Vậy số đơn ánh thỏa đề bài $C_{2 k}^k\left(C_{2 n}^{2 k+1}-C_{2 n-3}^{2 k}\right)$

Bài 3. Ta có $a(n)=\frac{n(n+1)}{2}, b(n)=\frac{n(n+1)(2 n+1)}{6}$
Khi đó $P(n)=2(n+1) a(n)+3 b(n)-1=\frac{n(n+1)(4 n+3)}{2}-1$.
Giả sử $P(n)$ là số chính phương ta có $n(n+1)(4 n+3)=2\left(x^2+1\right)$, ta có $n(n+1)(4 n+3)$ luôn có ước nguyên tố dạng $p=4 k+3$, suy ra $p \mid 2\left(x^2+1\right)$ suy ra $p|x, p| 1$, vô lí! Vậy không tồn tại $n$ để $P(n)$ là số chính phương.

Bài 4.

Ta tính được $\angle A=\frac{5 \pi}{8}, \angle B=\frac{\pi}{4}, \angle C=\frac{\pi}{8}$. Vẽ đường cao $A N, N$ thuộc $B C$.
Ta có $\frac{B N}{N C}=\frac{A N}{N C}=\frac{\sin C}{\cos C}$ và $\frac{A D}{C D}=\frac{A B}{B C}=\frac{\sin C}{\sin 5 C}, \sin 5 C=\cos C$, suy ra $\frac{B N}{N C}=\frac{A D}{C D}$, do đó $A N, B D, C M$ đồng quy tại $I$ và $D N | A B$.
Ta có $\angle B A N=\angle A N D=\angle A C K=2 \angle A C K$, suy ra $A C K$ cân và $N$ là trung điểm $A K$, từ đó tam giác $A B K$ vuông cân.
Khi đó $\angle F N K=\angle A C K=45^{\circ}=\angle A K B$ và $\angle A N M=45^{\circ}$, do đó $M, N, F$ thẳng hàng và $M F | B K$.

Đề thi và đáp án chọn đội dự tuyển 10 trường PTNK năm 2023

Bài 1. Cho ba số thực $a, b, c>0$ thỏa mãn $a+b+c=3$.
(a) Chứng minh rằng $(a b+b c+c a)(a b c+1) \geq 6 a b c$.
(b) Tìm số nguyên dương k lớn nhất sao cho $a b c\left(a^k+b^k+c^k\right) \leq 3$.

Bài 2 .Với mỗi số thực $x,[x]$ gọi là phần nguyên của $x$ – là số nguyên lớn nhất không vượt quá $x$ và ${x}:=x-[x]$ gọi là phần lẻ của $x$.
Cho $p$ là số nguyên tố lẻ, chứng minh rằng với mọi số nguyên dương $k$ nhỏ hơn $p$ thì tổng $$S=\left\{\frac{k}{p}\right\}+\left\{\frac{2 k}{p}\right\}+\left\{\frac{3 k}{p}\right\}+\ldots+\left\{\frac{(p-1) k}{p}\right\}$$ không đổi. Tính S.

Bài 3. Cho tam giác $A B C$ nôi tiếp đường tròn $(\omega)$, tiếp tuyến của $(\omega)$ tai $\mathrm{B}$ là $d_1$, tai $\mathrm{C}$ là $d_2$. I là điểm thuôc trung trự $\mathrm{BC}$, đường tròn tâm $\mathrm{I}$ bán kính $\mathrm{IB}$ cắt các canh $\mathrm{AB}, \mathrm{AC}$ tại $\mathrm{D}, \mathrm{E}$. $\mathrm{CD}$ cắt $d_1$ tai $\mathrm{F}, \mathrm{BE}$ cắt $d_2$ tai $\mathrm{G}$ sao cho $\mathrm{F}, \mathrm{G}$ cùng phía $\mathrm{A}$ so với $\mathrm{BC}$. Đường tròn ngoai tiếp tam giác $\mathrm{BDF}$ cắt $\mathrm{BE}$ tại $\mathrm{K}$, đường tròn ngoại tiếp tam giác CEG cắt $\mathrm{CD}$ tại L.
(a) Khi $\mathrm{I}$ thuộc $\mathrm{BC}$, gọi $\mathrm{P}$ là giao điểm của $\mathrm{FK}$ và $\mathrm{GL}$. Chứng minh $\mathrm{AP}$ đi qua tâm của $(\omega)$.
(b) Khi I khác phía $\mathrm{A}$ đối với $\mathrm{BC}, \mathrm{DE}$ cắt $d_1$ tại $\mathrm{R}, d_2$ tại $\mathrm{S}$. Đường tròn ngoại tiếp tam giác ISR cắt $\mathrm{BC}$ tại $\mathrm{X}, \mathrm{Y}$. Chứng minh $B X=C Y$.

Bài 4 Tìm số nguyên dương $s$ lớn nhất thỏa mãn tính chất sau: Với mọi bộ số nguyên dương nhỏ hơn hay bằng 10 (không nhất thiết phân biệt) có tồng bằng $s$ ta luôn có thể chia thành hai nhóm mà tổng các số thuộc mỗi nhóm nhỏ hơn hay bằng 70 .

Lời giải

Bài 1.

(a) Đặt $a=\min {a, b, c}$, suy ra $a \leq 1$.
Khi đó $(a-1)^3 \leq 1 \Rightarrow a^3-3 a^2+3 a-1 \leq 0 \Rightarrow \frac{1}{a}+a(3-a) \geq 3$, suy ra $\frac{1}{a}+a b+a c \geq 3$, hơn nữa
$$
\frac{1}{b}+\frac{1}{c}+b c \geq 3 \sqrt[3]{\frac{1}{b} \frac{1}{c} b c}=3
$$
Từ đó $\frac{1}{a}+\frac{1}{b}+\frac{1}{c}+a b+b c+a c \geq 6$. hay $(a b+b c+c a)(a b c+1) \geq 6 a b c$.
(b) Cho $a=2, b=c=\frac{1}{2}$, suy ra $k<3$, ta chứng minh $k=2$ thì bất đẳng thức thỏa với mọi $a, b, c$ thỏa điều kiện, thật vậy

$a b c\left(a^2+b^2+c^2\right) =\frac{1}{3} \cdot a b c(a+b+c)\left(a^2+b^2+c^2\right) $
$\leq \frac{1}{9} \cdot(a b+b c+c a)^2 \cdot\left(a^2+b^2+c^2\right) $
$=\frac{1}{9} \cdot(a b+b c+c a)(a b+b c+c a)\left(a^2+b^2+c^2\right) $
$ \leq \frac{1}{9} \cdot \frac{1}{27} \cdot\left(a^2+b^2+c^2+2(a b+b c+c a)\right)^3 $
$ =\frac{1}{9} \cdot \frac{1}{27} \cdot 3^6=3$

Bài 2.

Với $p$ nguyên tố lẻ thì $(k, p)=1$ với mọi $0<k<p$. Ta chứng minh $p-1$ số $k, 2 k, \cdots,(p-1) k$ là hệ thặng dư thu gọn của $p$, thật vậy, giả sử $i k \equiv j k($ $\bmod p)$ với $i, j<p$ thì $k(i-j) \equiv 0(\bmod p)$, suy ra $i=j$.
Khi đó $S=\left\{\frac{k}{p}\right\}+\left\{\frac{2 k}{p}\right\}+\left\{\frac{3 k}{p}\right\}+\ldots+\left\{\frac{(p-1) k}{p}\right\}=\frac{1}{p}+\frac{2}{p}+\cdots \frac{p-1}{p}=$ $\frac{p-1}{2}$ không đổi.

Bài 3.

(a) Gọi $O$ là tâm của $\omega$. Ta có $\angle S D B=\angle A D E=\angle A C B=\angle S B D$ nên $\triangle S B D$ cân tại $S$. Tương tự $\triangle R E C$ cân tại $R$. Biến đổi góc
$$
\angle K F L=\angle K F D=\angle K B D=\angle D C E=\angle E G L \angle K G L,
$$
suy ra $F, K, L, G$ đồng viên.
Do $I \in B C$ nên $\angle B D C=90^{\circ}$, mà $\triangle S B D$ cân tại $S$ nên $S$ là tâm đường tròn $(F D K)$. Tương tự, $R$ là tâm đường tròn $(G E L)$. Ta có
$$
A D \cdot A B=A E \cdot A C, \quad P K \cdot P F=P L \cdot P G,
$$
suy ra $A P$ là trục đẳng phương của hai đường tròn $(F D K)$ và $(G E L)$, do đó $A P \perp R S$.
Mà $A O \perp D E$ nên $A, O, P$ thằng hàng.

(b) Gọi $M, N$ lần lượt là giao điểm của $I S, I R$ với $B C . \triangle S B D$ cân tại $S$ nên suy ra $I S$ là đường trung trực của $B D$, tương tự $I R$ là đường trung
Tập san Toán học STAR EDUCATION
trực của $E C$. Biến đổi góc
$$
\begin{aligned}
& \angle M S D=90^{\circ}-\angle S D B=90^{\circ}-\angle A D E=90^{\circ}-\angle A C B=\angle C N G . \
\Rightarrow & \angle I S R=\angle Y N G \Rightarrow \angle I S Y+\angle Y S R=\angle M Y I+\angle Y I R \Rightarrow \angle I S Y= \
& \angle X Y I=\angle X S I .
\end{aligned}
$$
Vậy $S I$ là tia phân giác của $\angle X S Y$ nên $I$ nằm trên đường trung trực của $X Y$. Mà $I$ cũng nằm trên đường trung trực của $B C$ nên $B X=C Y$.

Bài 4.

Ta chứng minh rằng $s=133$ là số lớn nhất thoả mãn điều kiện bài toán. Trước hết, giả sử rằng $s$ là một số thoả mãn điều kiện đã cho.

Viết $s=9 k+r (k, r \in \mathbb{Z}{\geq 0}, 1 \leq r \leq 9 )$.

Nếu $s \geq 134$, xét một bộ số gồm $k$ số 9 và số còn lại bằng $s-9 k$. Trong bộ số này có không quá một số khác 9 nên khi chia chúng thành hai phần khác rỗng, phải có ít nhất một bộ chứa toàn số 9. Hơn nữa, $$ 9 \cdot 7=63<70<9 \cdot 8 $$ nên bộ số này có tổng tối đa là 63 . Nhưng khi đó tổng của các số còn lại, gọi là $T$, sẽ phải thoả mãn $$ T \geq 134-63=71>70 $$ vô lý do $T \leq 70$. Từ đó phải có $s \leq 133$. Bây giờ ta chứng minh rằng $s=133$ thoả mãn điều kiện bài toán. Trước hết, ta chứng minh rằng với mọi bộ số nguyên dương không vượt quá 10 có tổng bằng 133, khi chia thành hai phần khác rỗng là $X, Y$ khác rống (có thể có các phần tử trùng nhau), sao cho

$$ M=\sum{x \in X} x-\sum_{y \in Y} y \geq 0$$

và $M$ nhỏ nhất có thể, thì $M \leq 8$. Thật vậy, giả sử rằng $M \geq 9$ thì
$$
\sum_{x \in X} \geq \frac{1}{2}\left(\sum_{x \in X} x+\sum_{y \in Y} y+9\right) \geq \frac{133+9}{2}=71 .
$$
Vì mỗi phần tử của $X$ không vượt quá 10 nên $X$ có ít nhất 8 phần tử. Đặt $t=\min X$. Xét hai tập hợp
$$
\left\{\begin{array}{l}
X^{\prime}=X \cup{t} \
Y^{\prime}=Y \backslash{t}
\end{array}\right.
$$
thì $X^{\prime}, Y^{\prime} \neq \emptyset$, đều gồm các số nguyên dương không vượt quá 10 , và có tổng bằng 133. Vì tính nhỏ nhất của $M$ nên
$$
M \leq\left|\sum_{x \in X^{\prime}} x-\sum_{y \in Y^{\prime}} y\right|=\left|\sum_{x \in X} x-\sum_{y \in Y} y-2 t\right|=|M-2 t|
$$
Kết hợp với $M \geq 9$ và $1 \leq t \leq 10$ thì $9 \leq M \leq t \leq 10$. Có hai khả năng sau:

  • Nếu $M=10$ thì
    $$
    \sum_{x \in X} x=\frac{133+10}{2} \notin \mathbb{Z}
    $$
    là một điều vô lý.
  • Nếu $M=9$ thì
    $$
    \sum_{x \in X} x=\frac{133+9}{2}=71 .
    $$
    Nếu $t=9$ thì $X$ gồm toàn số 9 và số 10 , nên có thể viết được
    $$
    71=9 k+10 l\left(k, l \in \mathbb{Z}{\geq 0}\right) . $$ Do đó $9 k \equiv 1(\bmod 10)$, dẫn đến $k \equiv 9(\bmod 10)$ và $k \geq 9$. Hệ quả là $$ 9 k+10 l \geq 9 k \geq 81>71 $$ cũng là điều vô lý. Từ đó điều giả sử là sai hay phải có $M \leq 8$, dẫn đến $$ \sum{y \in Y} y \leq \sum_{x \in X} x \leq \frac{1}{2}\left(\sum_{x \in X}+\sum_{y \in Y} y+8\right)=\frac{133+8}{2} .
    $$
    Nhưng các tổng là số nguyên nên
    $$
    \sum_{y \in Y} y \leq \sum_{x \in X} x \leq 70,
    $$
    nghĩa là cách chia $(X, Y)$ thoả mãn điều kiện bài toán. Tóm lại, $s=133$ là số lớn nhất thoả mãn yêu cầu đề bài. Bài toán kết thúc.

ĐỀ THI CHỌN ĐỘI TUYỂN QUỐC GIA CỦA CÁC TỈNH, THÀNH

ĐỀ THI CHỌN ĐỘI TUYỂN TRƯỜNG PHỔ THÔNG NĂNG KHIẾU

ĐỀ THI CHỌN ĐỘI TUYỂN HSG QUỐC GIA CỦA TRƯỜNG PTNK NĂM 2008 – 2009 – Toán Việt (toanviet.net)

ĐỀ THI CHỌN ĐỘI TUYỂN HSG QUỐC GIA CỦA TRƯỜNG PTNK NĂM 2009 – 2010 – Toán Việt (toanviet.net)

ĐỀ THI CHỌN ĐỘI TUYỂN HSG QUỐC GIA CỦA TRƯỜNG PTNK 2010 – 2011 – Toán Việt (toanviet.net)

ĐỀ THI CHỌN ĐỘI TUYỂN HSG QUỐC GIA CỦA TRƯỜNG PTNK NĂM 2011 – 2012 – Toán Việt (toanviet.net)

ĐỀ THI CHỌN ĐỘI TUYỂN QUỐC GIA CỦA TRƯỜNG PTNK NĂM 2013 – 2014 – Toán Việt (toanviet.net)

ĐỀ THI CHỌN ĐỘI TUYỂN HSG QUỐC GIA CỦA TRƯỜNG PTNK NĂM 2014 – 2015 – Toán Việt (toanviet.net)

Đáp án thi chọn đội tuyển Toán trường PTNK năm 2015 – Toán Việt (toanviet.net)

Đáp án đề thi chọn đội tuyển trường Phổ thông Năng khiếu thi HSG QG năm 2016 – Toán Việt (toanviet.net)

ĐỀ THI CHỌN ĐỘI TUYỂN HSG QUỐC GIA CỦA TRƯỜNG PTNK NĂM 2017 – 2018 – Toán Việt (toanviet.net)

ĐỀ THI CHỌN ĐỘI TUYỂN HSG QUỐC GIA CỦA TRƯỜNG PTNK NĂM 2018 – 2019 – Toán Việt (toanviet.net)

ĐỀ THI CHỌN ĐỘI TUYỂN HSG QUỐC GIA CỦA TRƯỜNG PTNK NĂM 2019 – 2020 – Toán Việt (toanviet.net)

Đáp án đề thi chọn đội tuyển trường PTNK năm 2020 – Toán Việt (toanviet.net)

Đề thi và đáp án chọn đội tuyển toán trường PTNK năm 2021 – Toán Việt (toanviet.net)

ĐỀ THI CHỌN ĐỘI TUYỂN CÁC TỈNH THÀNH KHÁC

Hệ phương trình ba ẩn

Trong các bài trước mình đã làm quen với các hệ phương trình hai ẩn, phương pháp chủ yếu cũng là thế, cộng đại số, đặt ẩn phụ. Trong bài này chúng ta tiếp tục với các hệ phương trình nhiều ẩn hơn, chủ yếu là các hệ phương trình ba ẩn, trong các hệ phương trình này có hai dạng ta quan tâm và xuất hiện nhiều là hệ đối xứng và hệ hoán vị vòng quanh.

Hệ ba ẩn đối xứng

Hệ đối xứng ba biến là hệ có dạng

$\left\{\begin{array}{l}
f(x,y,z)=0 \\\\
g(x,y,z)=0 \\\\
h(x,y,z)=0
\end{array}\right.$

trong đó $f, g, h$ là các biểu thức đối xứng với $x, y, z$ tức là khi ta hoán vị $x, y, z$ thì $f, g, h$ vẫn không đổi.

Các biểu thức đối xứng 3 biến cơ bản nhất là $x+y+z, xy+yz+xz, xyz$.

Từ đó ta xét ví dụ sau

Ví dụ 1. Giải hệ phương trình $\left\{\begin{array}{l}
x+y+z=6 (1)\\\\
xy+yz+xz=11 (2)\\\\
xyz=6 (3)
\end{array}\right.$

Lời giải

Từ (1) ta có $y +z = 6-z$, từ (2), $ yz = 11-x(y+z) = 11 – x(6-x) = x^2-6x+11$.

Thế vào (3) ta có $x(x^2-6x+11) = 6$ $\Leftrightarrow x^3 -6x^2+ 11x – 6 = 0$

Giải ra được $x = 1, x = 2, x= 3$.

Với $x = 1$ ta có $y+z = 5, yz = 6$ giải ra được $y = 2, z= 3$ và $y=3, z=2$.

Các trường hợp khác tương tự, hệ phương trình có nghiệm $(1, 2, 3)$ và các hoán vị.

Do đó nếu hệ phương trình ba ẩn đối xứng, có một cách giải là ta tìm được giá trị của các biểu thức đối xứng cơ bản như bài trên.

Ví dụ 2. (PTNK Chuyên toán 2010) Giải hệ phương trình $\left\{\begin{array}{l}
x+y+z=3 \\\\
x y+y z+x z=-1 \\\\
x^3+y^3+z^3+6=3\left(x^2+y^2+z^2\right)
\end{array}\right.$

Lời giải

Ta chỉ cần tính được $xyz$ thì có thể đưa về ví dụ 1.

Từ (1) và (2) ta tính được $x^2+y^2+z^2 = (x+y+z)^2 – 2(xy+yz+xz) = 11$

Suy ra $x^3+y^3+z^3 = 27$

Mà $x^3+y^3+z^3 – 3xyz=(x+y+z)(x^2+y^2+z^2-xy-yz-xz) \Rightarrow xyz = -3$

do đó ta có $x+y+z = 3, xy+yz+xz = -1, xyz = -3$ tương tự ví dụ 1, ta giải được nghiệm là $(1,-1,3)$ và các hoán vị.

Ngoài cách trên ta có thể giải như sau

$x^3+y^3+z^3 = (x+y+z)^3 – 3(x+y)(y+z)(x+z)$, khi đó $(x+y)(y+z)(z+x) = 0$, tổng hai số bằng 0, ta suy ra số còn lại bằng 3, tiếp tục ta cũng có kết quả như trên.

Hệ hoán vị vòng quanh

Các hệ phương trình nhiều ẩn thường gặp là hệ hoán vị vòng quanh có dạng sau:

Phương pháp thường dùng là cộng đại số,phân tích thành tích, sử dụng đánh giá bất đẳng thức để chứng minh $x=y=z$.

Ta xét một số ví dụ sau:

Ví dụ 3. Giải hệ phương trình $\left\{\begin{array}{l}(x-y)^2=2 z-z^2(1) \\\\(y-z)^2=2 x-x^2(2)\\\\ (z-x)^2=2 y-y^2(3)\end{array}\right.$

Lời giải Lấy (1) trừ (2) ta có:

$(x-2 y+z)(x-z)=x^2-z^2-2(x-z)=(x-z)(x+z-2) \Leftrightarrow 2(x-z)(y-1)= 0$

$\Leftrightarrow x=z$ hoặc $y=1$
– $y=1$ ta có $(3) \Leftrightarrow(x-z)^2=1 \Leftrightarrow z=x+1, z=x-1$
+ $z=x+1$ giải được $ x=0, z=1$ và $x=1, z=2 $Khi đó ta có nghiệm $(0,1,1),(1,1,2)$
+ $z=x-1 $ giải ra được $x=1, z=0 $ và $ x=2, z=1 $Ta có nghiệm $(1,1,0)$ và $(2,1,1)$
Với $x=z$ từ (3) ta có $ y^2-2 y=0 \Leftrightarrow y=0, y=2$

Với $y=0$ ta có $\left\{\begin{array}{l}x^2=2 z-z^2 \\\\ z^2=2 x-x^2\end{array} \Leftrightarrow \left\{\begin{array}{l}2 z^2=2 z \\\\ x-z\end{array}\right.\right.$.

Giải được nghiệm $(0,0,0)$ và $(1,0,1)$.

+Với $y=2$, giải ra được nghiệm $(1,2,1)$ và $(2,2,2)$. Vậy hệ phương trình có 8 nghiệm.

Ví dụ 4. (PTNK Chuyên Toán 2103) Giải hệ phương trình $\left\{\begin{array}{l}
3 x^2+2 y+1=2 z(x+2) \\\\
3 y^2+2 z+1=2 x(y+2) \\\\
3 z^2+2 x+1=2 y(z+2)
\end{array}\right.$

Lời giải Cộng ba phương trình lại ta có:
$3\left(x^2+y^2+z^2\right)+2(x+y+z)+3=2(x y+y z+z x)+4(x+y+z) $

$ \Leftrightarrow 3\left(x^2+y^2+z^2\right)-2(x y+y z+x z)-2(x+y+z)+3=0 $
$\Leftrightarrow(x-y)^2+(y-z)^2+(z-x)^2+(x-1)^2+(y-1)^2+(z-1)^2=0 $
$\Leftrightarrow\left\{\begin{array}{l}
x=1 \\\\
y=1 \\\\
z=1
\end{array}\right.
$
Thử lại thấy $(1,1,1)$ là nghiệm của hệ.

Ví dụ 5. Giải hệ phương trình $\left\{\begin{array}{l}
2 x=y^2-z^2 \\\\
2 y=z^2-x^2 \\\\
2 z=x^2-y^2
\end{array}\right.$

Lời giải

Lấy (1) $+(2)$ ta có $(x+y)(x-y+2)=0 \Leftrightarrow x+y=0$ hoặc $x=2-y$.
Với $x+y=0$, từ (3) ta có $z=0$, từ (1) ta có $x=0$ hoặc $x=2$. Ta có nghiệm $(x, y, z)$ là $(0,0,0)$ và $(2,-2,0)$.
Với $x=y-2$, từ (3) ta có $2 z=(y-2)^2-y^2=4-4 y \Leftrightarrow z=2-2 y$. Thế vào (1) ta có: $2(y-2)=y^2-(2-2 y)^2 \Leftrightarrow y^2-2 y=0 \Leftrightarrow y=0, y=2$. Từ đó ta có nghiệm $(-2,0,2)$ và $(2,-2,0)$. Vậy hệ có 4 nghiệm.

Hệ nhiều ẩn không mẫu mực

Một số hệ không mẫu mực thì không có cách giải chung, do đó ta phải để đặc điểm của các hệ phương trình này để có cách giải phù hợp, chủ yếu cũng là giảm được ẩn, phân tích nhân tử, . ..

Ví dụ 6. Giải hệ phương trình sau: $\left\{\begin{array}{l}
(x-2 y)(x-4 z)=55 \\\\
(y-2 z)(y-4 x)=-39 \\\\
(z-2 x)(z-4 y)=-16
\end{array}\right.$

Lời giải

$\left\{\begin{array}{l}(x-2 y)(x-4 z)=55 \\\\ (y-2 z)(y-4 x)=-39 \\\\ (z-2 x)(z-4 y)=-16\end{array} \Leftrightarrow\left\{\begin{array}{l}x^2-2 x y-4 x z+8 y z=55(1) \\\\ y^2-2 y z-4 x y+8 x z=-39(2) \\\\ z^2-2 x z-4 y z+8 x y=-16(3)\end{array}\right.\right.$

Cộng (1),(2),(3) ta có $(x+y+z)^2=0 \Leftrightarrow x+y+z=0 \Leftrightarrow z=-x-y$
Thế vào (1),(2) ta có $\left\{\begin{array}{l}(x-2 y)(5 x+4 y)=55 \\\\ (3 y+2 x)(y-4 x)=-39\end{array}\right.$

$\Leftrightarrow\left\{\begin{array}{l}5 x^2-6 x y-8 y^2=55 \\\\ 3 y^2-10 x y-8 x^2=-39\end{array}\right.$
Nhận thấy $y=0$ không thỏa hpt:
Đặt $x=k y$, ta có hệ

$\left\{\begin{array}{l}
y^2\left(5 k^2-6 k-8\right)=55 \\\\
y^2\left(-8 k^2-10 k+3\right)=-39
\end{array}\right. $
$\Rightarrow-39\left(5 k^2-6 k-8\right)=55\left(-8 k^2-10 k+3\right) $
$\Leftrightarrow 245 k^2+784 k+147=0$
$ \Leftrightarrow\left[\begin{array}{l}
k=-3 \\\\
k=\frac{-1}{5}
\end{array}\right.
$
Với $k=-3$, ta có $y=1$, hoặc $y=-1$. Từ đó ta có nghiệm là $(-3,1,2),(3,-1,-2)$
Với $k=-\frac{1}{5}$ (vô nghiệm)

Chìa khóa trong lời giải này chính là đặc điểm của các hệ số tự do bên phải của các phương trình.

Qua một số ví dụ , hi vọng các em rút ra kinh nghiệm trong việc giải một số hệ phương trình nhiều ẩn, cùng rèn luyện các bài toán sau nhé.

Bài tập rèn luyện

Bài 1. Giải các hệ phương trình sau

1)$\begin{cases} x^2(y+z)^2=(3x^2+x+1)y^2z^2&\\\\y^2(z+x)^2=(4y^2+y+1)z^2x^2&\\\\z^2(x+y)^2=(5z^2+z+1)=x^2y^2 \end{cases}$ 2)$\left\{ \begin{array}{l}xy = x + 3y\\\\yz = 2\left( {y + z} \right)\\\\xz = 3\left( {3z + 2x} \right)\end{array} \right.$ 3) $\left\{ \begin{array}{l}
{\left( {x + y + z} \right)^3} = 12t\\\\
{\left( {y + z + t} \right)^3} = 12x\\\\
{\left( {z + t + z} \right)^3} = 12y\\\\
{\left( {t + x + y} \right)^3} = 12z
\end{array} \right.$

Bài 2. Giải hệ phương trình sau:

1)$\left\{\begin{array}{l}
x^{3}+x^{2}+x-2=y \\\\
y^{3}+y^{2}+y-2=z \\\\
z^{3}+z^{2}+z-2=x
\end{array}\right.$
2) $\left\{\begin{array}{l}
y^{3}-6 x^{2}+12 x-8=0 \\\\
z^{3}-6 y^{2}+12 y-8=0 \\\\
x^{3}-6 z^{2}+12 z-8=0
\end{array}\right.$
Bài 3. Giải hệ phương trình $\begin{cases}ab+c+d=3&\\\\bc+d+a=5&\\\\cd+a+b=2&\\\\da+b+c=6 \end{cases}$

Bài 4.

Cho $a \in \mathbb{R}$. Giải hệ phương trình $\begin{cases} x_1^2+ax_1+(\dfrac{a-1}{2})^2=x_2&\\\\
x_2^2+ax_2+(\dfrac{a-1}{2})^2=x_3&\
…&\\\\
x_n^2+ax_n+(\dfrac{a-1}{2})^2=x_1
\end{cases}$

Hệ phương trình chứa tham số

Hệ phương trình và các phương pháp giải của nó chúng ta đã nghiên cứu trong các bài giảng trước, bài viết này ta tiếp tục với các hệ phương trình nhưng chứa thêm tham số, việc giải các hệ phương trình chứa tham số căn bản cũng dựa trên các phương pháp đã biết, tuy vậy ta phải xét nhiều trường hợp hơn đòi hỏi suy luận tốt và sự cẩn thận nhất định của học sinh.

Ví dụ 1. Cho hệ phương trình: $\left\{\begin{array}{l}\sqrt{x-2}+\sqrt{y-1}=2 \\\\ x+y=m\end{array}\right.$
(a) Giải hệ với $m=7$
(b) Tìm $m$ sao cho hệ có nghiệm $(x, y)$

Lời giải
a) $\left\{\begin{array}{l}\sqrt{x-2}+\sqrt{y-1}=2 \\\\ x+y=m\end{array}\right.$
ĐKXĐ: $x \geq 2, y \geq 1$
Đặt $ a=\sqrt{x-2}, b = \sqrt{y-1}$ ta có $a, b \geq 1$ và $a+b = 2, a^2+b^2 = 4$.

Từ đó ta có $b = 2-a, a^2+(2-a)^2 = 4$, giải ra được $a= 2, b=0$ và $a=0, b=2$.

Với $a = 2,b=0$ ta có $x=6, y=1$

Với $a=0,b=2$ ta có $x=2, y = 5$.

Vậy hệ phương trình có hai nghiệm $(2 ; 5),(6 ; 1)$

b) Đặt $u=\sqrt{x-2}, v=\sqrt{y-1}(u, v \geq 0$
Hệ phương trình trở thành: $\left\{\begin{array}{l}u+v=2 \\\\ u^2+v^2=m-3\end{array}\right.$ $\Rightarrow 2 u^2-4 u+7-m=0 \quad(2)$
Để hệ (1) có nghiệm thì (2) phải có nghiệm không âm, nhỏ hơn hoặc bằng 2, khi và chỉ khi:
$$
\left\{\begin{array} { l }
{ \Delta ^ { \prime } \geq 0 } \\\\
{ S > 0 } \\\\
{ P \geq 0 } \\\\
{ ( x _ { 1 } – 2 ) ( x _ { 2 } – 2 ) > 0 } \\\\
{ S \leq 4 }
\end{array} \Leftrightarrow \left\{\begin{array}{l}
m \geq 5 \\\\
m \leq 7
\end{array}\right.\right.
$$
Vậy $5 \leq m \leq 7$ thì hệ đã cho có nghiệm $(x, y)$

Ví dụ 2. Giải và biện luận hệ phương trình sau: $\left\{\begin{array}{l}\frac{x y z}{x+y}=m \\\\ \frac{x y z}{y+z}=1 \ \frac{x y z}{z+x}=2\end{array}\right.$

Lời giải

Lời giải. Đặt $a=x y, b=y z, c=x z$ ta tính được: $\frac{1}{a}=\frac{3 m-2}{4 m}, \frac{1}{b}=\frac{m+2}{4 m}, \frac{1}{c}=\frac{2-m}{4 m}$.
Khi đó $\frac{1}{(x y z)^2}=\frac{1}{a b c}=\frac{(3 m-2)(m+2)(2-m)}{64 m^3}=P$.
Nếu $P \leq 0 \Leftrightarrow m \leq-2,0 \leq m \leq \frac{2}{3}$ hoặc $m \geq 2$ thì hệ vô nghiệm.
Ta có $P>0 \Leftrightarrow-2<m<0$ hoặc $\frac{2}{3}<m<2$.
Khi đó $(x y z)^2=\frac{64 m^3}{(3 m-2)(m+2)(2-m)}=\frac{1}{P}$. Suy ra $x y z= \pm \sqrt{\frac{1}{P}}$.

  • Nếu $x y z=\sqrt{\frac{1}{P}}$ thì $x=\frac{2-m}{4 m} \sqrt{\frac{1}{P}}$,
    $$
    y=\frac{m+2}{4 m} \sqrt{\frac{1}{P}}, z=\frac{3 m-2}{4 m} \sqrt{\frac{1}{P}} \text {. }
    $$
  • Nếu $x y z=-\sqrt{\frac{1}{P}}$ thì $x=\frac{m-2}{4 m} \sqrt{\frac{1}{P}}$,
    $$
    y=\frac{-m-2}{4 m} \sqrt{\frac{1}{P}}, z=\frac{2-3 m}{4 m} \sqrt{\frac{1}{P}} \text {. }
    $$

Ví dụ 3. Cho hệ phương trình $\left\{\begin{array}{l}(x-2 y)(x+m y)=m^2-2 m-3 \\\\ (y-2 x)(y+m x)=m^2-2 m-3\end{array}\right.$

a) Giải hệ phương trình khi $m=-3$

b) Tìm $m$ để hệ có ít nhất một nghiệm $\left(x_\circ, y_\circ \right)$ thỏa $x_\circ>0, y_\circ>0$.

Lời giải
a) Khi $m=-3$ ta có hệ:
$$
\left\{\begin{array} { l }
{ ( x – 2 y ) ( x – 3 y ) = 1 2 } \\\\
{ ( y – 2 x ) ( y – 3 x ) = 1 2 }
\end{array} \Leftrightarrow \left\{\begin{array}{l}
x^2-5 x y+6 y^2=12(1) \\\\
y^2-5 x y+6 x^2=12(2)
\end{array}\right.\right.
$$
Lấy (1) – (2) ta có $5\left(y^2-x^2\right)=0 \Leftrightarrow x=y, x=-y$.
Với $x=y$ thế vào (1) ta có $x^2=6 \Leftrightarrow x=\sqrt{6}, y=\sqrt{6}$ hoặc $x=-\sqrt{6}, y=$ $-\sqrt{6}$
Với $x=-y$ thế vào (1) ta có $x^2=1 \Leftrightarrow x=1, x=-1$. Với $x=1, y=-1$, với $x=-1, y=1$.
Vậy hệ phương trình có 4 nghiệm.
b) Hệ có thể viết lại $\left\{\begin{array}{l}x^2+(m-2) x y-2 m y^2=m^2-2 m-3(1) \\\\y^2+(m-2) x y-2 m x^2=m^2-2 m-3(2)\end{array}\right.$
Lấy (1) – (2) ta có $(2 m+1)\left(y^2-x^2\right)=0$.
Xét $m=\frac{-1}{2}$ ta có hệ trở thành: $x^2-\frac{5}{2} x y+y^2+\frac{7}{4}=0$, có nghiệm $\left(\frac{5+\sqrt{2}}{2}, 2\right)$ thỏa đề bài.
Xét $m \neq \frac{-1}{2}$ ta có $x=y$ hoặc $x=-y$. Trường hợp $x=-y$ không thỏa đề bài.
Trường hợp $x=y$, thế vào (1) ta có:
$$
-(m+1) x^2=m^2-2 m-3=(m+1)(m-3)
$$
Nếu $m=-1$ ta có $(x-2 y)(x-y)=0,(y-2 x)(y-x)=0$ có nghiệm thỏa đề bài, chỉ cần chọn $x=1, y=1$.
Nếu $m \neq-1$ ta có $x^2=3-m$ để có nghiệm $x_o=y_o>0$ thì $m<3$. Khi đó phương trình có nghiệm $x_0=\sqrt{3-m}, y_o=\sqrt{3-m}$ thỏa đề bài.
Kết luận $m=\frac{-1}{2}, m=-1$ và $m<3$.

Ví dụ 4. Cho hệ phương trình với $k$ là tham số:
$$\left\{\begin{array}{l}
\frac{x}{\sqrt{y z}}+\sqrt{\frac{x}{y}}+\sqrt{\frac{x}{z}}=k \\\\
\frac{y}{\sqrt{z x}}+\sqrt{\frac{y}{z}}+\sqrt{\frac{y}{x}}=k \\\\
\frac{z}{\sqrt{x y}}+\sqrt{\frac{z}{x}}+\sqrt{\frac{z}{y}}=k
\end{array}\right.
$$
(a) Giải hệ với $k=1$.
(b) Chứng minh hệ vô nghiệm với $k \geq 2$ và $k \neq 3$.

Lời giải

Điều kiện xác định là: $x, y, z$ cùng dương hoặc cùng âm.
Đặt $a=\sqrt{\frac{x}{y}}, b=\sqrt{\frac{y}{z}}, c=\sqrt{\frac{z}{x}}$ thì $a, b, c>0$ và $a b c=1$.
Ta có: $\frac{a}{c}=\frac{|x|}{\sqrt{y z}}, \frac{b}{a}=\frac{|y|}{\sqrt{z x}}, \frac{c}{b}=\frac{|z|}{\sqrt{x y}}$.
a) Khi $k=1$, nếu $x, y, z>0$ thì $\frac{a}{c}+a+\frac{1}{c}=\frac{b}{a}+b+\frac{1}{a}=\frac{c}{b}+c+\frac{1}{b}=1$.
Cộng lại suy ra $\left(a+\frac{1}{a}\right)+\left(b+\frac{1}{b}\right)+\left(c+\frac{1}{c}\right)+\left(\frac{c}{c}+\frac{b}{a}+\frac{c}{b}\right)=3$
Theo bất đẳng thức Cô-si thì rõ ràng $a+\frac{1}{a} \geq 2, b+\frac{1}{b} \geq 2, c+\frac{1}{c} \geq 2$ nên đẳng thức trên không thể xảy ra.
Xét trường hợp $x, y, z$ cùng âm thì
$$
-\frac{a}{c}+a+\frac{1}{c}=-\frac{b}{a}+b+\frac{1}{a}=-\frac{c}{a}+c+\frac{1}{b}=1
$$
Trừ vào các vế và phân tích, ta suy ra:
$$
\frac{(a-1)(b-1)}{a}=\frac{(b-1)(c-1)}{b}=\frac{(c-1)(a-1)}{c}=0
$$
Từ đây dễ dàng suy ra ít nhất 2 trong $a, b, c$ phải là 1 mà $a b c=1$ nên $a=b=c=1$. Vì thế nên thay vào ta có $x=y=z<0$. Và mọi bộ số như thế đều thỏa mãn hệ.

b) Với $k \geq 2$, giả sử hệ có nghiệm $(x, y, z)$. Nếu như $x, y, z<0$ thì ta có $\frac{(a-1)(b-1)}{a}=\frac{(b-1)(c-1)}{b}=\frac{(c-1)(a-1)}{c}=k-1>0$.
Từ đó suy ra $a-1, b-1, c-1$ dều cùng dấu, kéo theo $a, b, c>1$ hoặc $a, b, c<1$ Tuy nhiên $a b c=1$ nên điều này không thể xảy ra. Do đó, ta phải có $a, b, c>0$ nên đưa về
$$
\frac{a}{c}+a+\frac{1}{c}=\frac{b}{a}+b+\frac{1}{a}=\frac{c}{b}+c+\frac{1}{b}=k
$$
Trong các số $a, b, c$ giả sử $a=\max {a, b, c}$ thì $k=\frac{a}{c}+a+\frac{1}{c} \geq$ $\frac{a}{c}+2 \sqrt{\frac{a}{c}} \geq 1+2=3$ nên ta cần có $k \geq 3$. Vì $k \neq 3$ nên $k>3$.
Vì $a=\max {a, b, c} \geq 1$ nên ta có $2 b+1 \geq \frac{b}{a}+b+\frac{1}{a}=k>3$ kéo theo $b>1$. Tương tự từ $2 c+1>\frac{c}{b}+c+\frac{1}{b}=k>3$ nên $c>1$. Từ đây suy ra $a, b, c>1$ trong khi $a b c=1$, vô lý.
Vậy hệ luôn vô nghiệm với $k \geq 2$ và $k \neq 3$.

Bài tập rèn luyện

Bài 1. Cho hê phương trình $\left\{\begin{array}{l}x+y=m-2 \\\\x^2+y^2+2 x+2 y=-m^2+4\end{array}\right.$ (trong đó $m$ là tham số $x$ và y là ẩn)
a) Tìm $m$ để hệ phương trình trên có nghiệm.
b) Tìm giá trị lớn nhất, nhỏ nhất của biểu thúc $A=x y+2(x+y)+2011$.

Bài 2. Cho hệ phương trình $\left\{\begin{array}{c}x^2+y^2+x y=m^2-2 m+4 \\\\ x^2+y^2-3 x y=5 m^2-10 m+4\end{array} \quad\right.$ (m là tham số)
a) Giải hệ phương trình khi $m=-1$.
b) Chứng minh rằng hệ phương trình luôn có nghiệm với mọi giá trị của $m$. Tìm $m$ để phương trình có nghiệm $(x ; y)$ thỏa $y>x>0$ và $5 x^2-2 x y+y^2$ đạt giá trị nhỏ nhất.

Bài 3. Tìm $a$ để hệ phương trình
$\left\{\begin{array}{c}
& \frac{a x+y}{y+1}+\frac{a y+x}{x+1}=a \\\\
& a x^2+a y^2=(a-2) x y-x
\end{array} \quad\right.$
có nghiệm duy nhất.

Đề thi và đáp án chọn đội tuyển Phổ thông Năng khiếu và các tỉnh thành

Đề thi và đáp án thi chọn đổi tuyển trường PTNK – ĐHQG TPHCM

ĐỀ THI CHỌN ĐỘI TUYỂN HSG QUỐC GIA CỦA TRƯỜNG PTNK NĂM 2020 – 2021 – Toán Việt (toanviet.net)

ĐỀ THI CHỌN ĐỘI TUYỂN HSG QUỐC GIA CỦA TRƯỜNG PTNK NĂM 2019 – 2020 – Toán Việt (toanviet.net)

ĐỀ THI CHỌN ĐỘI TUYỂN HSG QUỐC GIA CỦA TRƯỜNG PTNK NĂM 2018 – 2019 – Toán Việt (toanviet.net)

ĐỀ THI CHỌN ĐỘI TUYỂN HSG QUỐC GIA CỦA TRƯỜNG PTNK NĂM 2016 – 2017 – Toán Việt (toanviet.net)

Đáp án đề thi chọn đội tuyển trường Phổ thông Năng khiếu thi HSG QG năm 2016 – Toán Việt (toanviet.net)

Đáp án thi chọn đội tuyển Toán trường PTNK năm 2015 – Toán Việt (toanviet.net)

ĐỀ THI CHỌN ĐỘI TUYỂN HSG QUỐC GIA CỦA TRƯỜNG PTNK NĂM 2014 – 2015 – Toán Việt (toanviet.net)

ĐỀ THI CHỌN ĐỘI TUYỂN QUỐC GIA CỦA TRƯỜNG PTNK NĂM 2013 – 2014 – Toán Việt (toanviet.net)

ĐỀ THI CHỌN ĐỘI TUYỂN HSG QUỐC GIA CỦA TRƯỜNG PTNK NĂM 2011 – 2012 – Toán Việt (toanviet.net)

ĐỀ THI CHỌN ĐỘI TUYỂN HSG QUỐC GIA CỦA TRƯỜNG PTNK 2010 – 2011 – Toán Việt (toanviet.net)

ĐỀ THI CHỌN ĐỘI TUYỂN HSG QUỐC GIA CỦA TRƯỜNG PTNK NĂM 2009 – 2010 – Toán Việt (toanviet.net)

ĐỀ THI CHỌN ĐỘI TUYỂN HSG QUỐC GIA CỦA TRƯỜNG PTNK NĂM 2008 – 2009 – Toán Việt (toanviet.net)

Đề thi và đáp án chọn đội tuyển của một số tỉnh thành

Đề thi học sinh giỏi khối 10

Kì thi chọn đội dự tuyển trường Phổ thông Năng khiếu

Đề thi và đáp án chọn đội dự tuyển 10 trường PTNK năm 2023

Đề thi và đáp án chọn đội dự tuyển trường PTNK năm 2022

Đề thi và đáp án chọn đội dự tuyển PTNK năm 2021 – Toán Việt (toanviet.net)

Đề thi và đáp án chọn đội dự tuyển trường PTNK năm 2020 – Toán Việt (toanviet.net)

Đề thi và đáp án chọn đội dự tuyển PTNK năm học 2019 – 2020 – Toán Việt (toanviet.net)

Đề và đáp thi chọn đội dự tuyển PTNK năm học 2017 – 2018 – Toán Việt (toanviet.net)

Đáp án đề thi chọn đội dự tuyển lớp 10 năm 2016 – 2017 – Toán Việt (toanviet.net)

Đề và đáp án thi chọn đội dự tuyển lớp 10 năm 2012 – 2013 – Toán Việt (toanviet.net)

Kì thi Olympic truyền thống 30/4 (SGD TPHCM)

ĐỀ THI OLYMPIC 30 THÁNG 4 – TOÁN LỚP 10 NĂM 2011 – Toán Việt (toanviet.net)

ĐỀ THI OLYMPIC 30 THÁNG 4 – TOÁN LỚP 10 NĂM 2010 – Toán Việt (toanviet.net)

ĐỀ THI OLYMPIC 30 THÁNG 4 – TOÁN LỚP 10 NĂM 2009 – Toán Việt (toanviet.net)

ĐỀ THI OLYMPIC 30 THÁNG 4 – TOÁN LỚP 10 NĂM 2008 – Toán Việt (toanviet.net)

ĐỀ THI OLYMPIC 30 THÁNG 4 – TOÁN LỚP 10 NĂM 2007 – Toán Việt (toanviet.net)

ĐỀ THI OLYMPIC 30 THÁNG 4 – TOÁN LỚP 10 NĂM 2005 – Toán Việt (toanviet.net)

ĐỀ THI OLYMPIC 30 THÁNG 4 – TOÁN LỚP 10 NĂM 2004 – Toán Việt (toanviet.net)

ĐỀ THI OLYMPIC 30 THÁNG 4 – TOÁN LỚP 10 NĂM 2003 – Toán Việt (toanviet.net)

ĐỀ THI OLYMPIC 30 THÁNG 4 – TOÁN LỚP 10 NĂM 2002 – Toán Việt (toanviet.net)

ĐỀ THI OLYMPIC 30 THÁNG 4 – TOÁN LỚP 10 NĂM 2000 – Toán Việt (toanviet.net)

ĐỀ THI OLYMPIC 30 THÁNG 4 – TOÁN LỚP 10 NĂM 1999 – Toán Việt (toanviet.net)

ĐỀ THI OLYMPIC 30 THÁNG 4 – TOÁN LỚP 10 NĂM 1998 – Toán Việt (toanviet.net)

Kì thi duyên hải Bắc bộ

Kì thi HSG lớp 10 của các tỉnh, thành phố

Đề thi tuyển sinh vào 10 chuyên toán

Trường Phổ thông Năng khiếu

Toán chung cho tất cả các thí sinh

Đáp án đề thi Toán không chuyên trường Phổ thông Năng Khiếu năm 2021 – Toán Việt (toanviet.net)

Đề thi và đáp án tuyển sinh vào lớp 10 PTNK không chuyên 2020 – Toán Việt (toanviet.net)

Đề thi và đáp án tuyển sinh vào lớp 10 PTNK không chuyên 2019 – Toán Việt (toanviet.net)

Đề thi và đáp án tuyển sinh vào lớp 10 PTNK không chuyên 2018 – Toán Việt (toanviet.net)

Đề thi và đáp án tuyển sinh vào lớp 10 PTNK không chuyên 2017 – Toán Việt (toanviet.net)

Đề thi và đáp án tuyển sinh vào lớp 10 PTNK không chuyên 2016 – Toán Việt (toanviet.net)

Đề thi và đáp án tuyển sinh vào lớp 10 PTNK không chuyên 2015 – Toán Việt (toanviet.net)

Đề thi và đáp án tuyển sinh vào lớp 10 PTNK không chuyên 2014 – Toán Việt (toanviet.net)

Đề thi và đáp án tuyển sinh vào lớp 10 PTNK không chuyên 2013 – Toán Việt (toanviet.net)

Đề thi và đáp án tuyển sinh vào lớp 10 PTNK không chuyên 2012 – Toán Việt (toanviet.net)

Đề thi và đáp án tuyển sinh vào lớp 10 PTNK không chuyên 2011 – Toán Việt (toanviet.net)
Môn toán chuyên
ĐỀ THI VÀO LỚP 10 CHUYÊN TOÁN TRƯỜNG PHỔ THÔNG NĂNG KHIẾU 2021 – Toán Việt (toanviet.net)

Đề thi vào lớp 10 Chuyên Toán vào trường PTNK năm 2020 – Toán Việt (toanviet.net)

ĐỀ THI VÀO LỚP 10 CHUYÊN TOÁN PHỔ THÔNG NĂNG KHIẾU 2019 – Toán Việt (toanviet.net)

ĐỀ THI VÀO LỚP 10 CHUYÊN TOÁN PHỔ THÔNG NĂNG KHIẾU 2018 – Toán Việt (toanviet.net)

ĐỀ THI VÀO LỚP 10 CHUYÊN TOÁN TRƯỜNG PHỔ THÔNG NĂNG KHIẾU 2017 – Toán Việt (toanviet.net)

Đề thi vào lớp 10 chuyên toán Phổ thông Năng khiếu: Năm 2016 – Toán Việt (toanviet.net)

ĐỀ THI VÀO LỚP 10 TRƯỜNG PHỔ THÔNG NĂNG KHIẾU 2015 – Toán Việt (toanviet.net)

ĐỀ THI VÀO LỚP 10 CHUYÊN TOÁN TRƯỜNG PHỔ THÔNG NĂNG KHIẾU 2014 – Toán Việt (toanviet.net)

Đáp án và bình luận thi vào lớp 10 PTNK năm 2013: Đề chuyên toán – Toán Việt (toanviet.net)

ĐỀ THI VÀO LỚP 10 CHUYÊN TOÁN TRƯỜNG PHỔ THÔNG NĂNG KHIẾU 2012 – Toán Việt (toanviet.net)

SGD TP. Hồ Chí Minh

Đề toán chung cho tất cả các thí sinh

Đề thi và đáp án tuyển sinh vào 10 TPHCM 2020 – Toán Việt (toanviet.net)

Đề thi và đáp án tuyển sinh vào 10 TPHCM 2019 – Toán Việt (toanviet.net)

Đề thi và đáp án tuyển sinh vào 10 TPHCM 2018 – Toán Việt (toanviet.net)

Đề thi và đáp án vào lớp 10 TPHCM 2017 – Toán Việt (toanviet.net)

Đề thi và đáp án tuyển sinh vào lớp 10 TPHCM 2016 – Toán Việt (toanviet.net)

Đề thi và đáp án tuyển sinh vào lớp 10 TPHCM 2015 – Toán Việt (toanviet.net)

Đề thi và đáp án thi vào lớp 10 TPHCM 2014 – Toán Việt (toanviet.net)

Đề thi và đáp án thi vào lớp 10 TPHCM 2013 – Toán Việt (toanviet.net)

Đề thi và đáp án tuyển sinh vào lớp 10 TPHCM 2012 – Toán Việt (toanviet.net)

Đề thi và đáp án tuyển sinh vào 10 TPHCM 2011 – Toán Việt (toanviet.net)

Đề toán chuyên

ĐỀ THI VÀO LỚP 10 CHUYÊN TOÁN TP.HCM NĂM 2020 – Toán Việt (toanviet.net)

ĐỀ THI VÀO LỚP 10 CHUYÊN TOÁN TP.HCM NĂM 2019 – Toán Việt (toanviet.net)

ĐỀ THI VÀO LỚP 10 CHUYÊN TOÁN TP.HCM NĂM 2018 – Toán Việt (toanviet.net)

ĐỀ THI VÀO LỚP 10 CHUYÊN TOÁN TP.HCM NĂM 2017 – Toán Việt (toanviet.net)

ĐỀ THI VÀO LỚP 10 CHUYÊN TOÁN TP.HCM NĂM 2016 – Toán Việt (toanviet.net)

ĐỀ THI VÀO LỚP 10 CHUYÊN TOÁN TP.HCM – NĂM 2015 – Toán Việt (toanviet.net)

ĐỀ THI VÀO LỚP 10 CHUYÊN TOÁN TP.HCM – NĂM 2014 – Toán Việt (toanviet.net)

ĐỀ THI VÀO LỚP 10 CHUYÊN TOÁN TP.HCM 2013 – Toán Việt (toanviet.net)

ĐỀ THI VÀO CHUYÊN TOÁN LỚP 10 TP.HCM 2012 – Toán Việt (toanviet.net)

Đề thi thử Star Education

Đề toán chung

ĐỀ THI THỬ VÀO LỚP 10 KHÔNG CHUYÊN – TT STAR EDUCATION 2022 – Toán Việt (toanviet.net)

ĐỀ THI THỬ VÀO LỚP 10 KHÔNG CHUYÊN LẦN 2 TT STAR EDUCATION 2020 – Toán Việt (toanviet.net)

Đề thi thử vào lớp 10 – Không chuyên PTNK – Toán Việt (toanviet.net)

Đề thi thử vào lớp 10 PTNK – Đề toán chung – Lần 2 – Toán Việt (toanviet.net)

Đề toán chuyên

ĐỀ THI THỬ VÀO LỚP 10 TOÁN CHUYÊN – TT STAR EDUCATION 2022 – Toán Việt (toanviet.net)

Đề thi thử vào lớp chuyên toán Star Education năm 2021 – Lần 2 – Toán Việt (toanviet.net)

ĐỀ THI THỬ VÀO LỚP 10 TRUNG TÂM STAR EDUCATION TOÁN CHUYÊN – 2020 – Toán Việt (toanviet.net)

Đề thi và đáp án kì thi chọn đội tuyển thi Quốc gia trường Phổ thông Năng khiếu năm học 2018 – 2019

ĐỀ THI

Ngày thi thứ nhất

Bài 1. Cho số nguyên $a>1$. Tìm giá trị lớn nhất của số thực $d$ sao cho tồn tại một cấp số cộng có công sai $d$, số hạng đầu tiên là $a$ và có đúng hai trong các số $a^2, a^3, a^4, a^5$ là những số hạng của cấp số cộng đó.

Bài 2. Cho $n$ số thực $x_1, x_2, \ldots, x_n$. Với mỗi $i \in{1,2, \ldots, n}$, gọi $a_i$ là số các chỉ số $j$ mà $\left|x_i-x_j\right| \leq 1$ và $b_i$ là số các chỉ số $j$ mà $\left|x_i-x_j\right| \leq 2$ ( $i$ và $j$ có thể bằng).

(a) Chứng minh rằng tồn tại $i$ để $b_i \leq 3 a_i$.

(b) Gọi $A$ là số cặp $(i, j)$ có thứ tự mà $\left|x_i-x_j\right| \leq 1$ và $B$ là số cặp $(i, j)$ có thứ tự mà $\left|x_i-x_j\right| \leq 2$ ( $i$ và $j$ có thể bằng nhau). Chứng minh rằng $B \leq 3 A$.

Bài 3. Cho $p$ là số tự nhiên. Xét phương trình nghiệm nguyên $x^3+x+p=y^2$.

(a) Tìm số nguyên tố $p$ nhỏ nhất dạng $4 k+1$ sao cho phương trình có nghiệm.

(b) Chứng minh rằng nếu $p$ là số chính phương thì phương trình trên có nghiệm nguyên dương.

Bài 4. Cho tam giác $A B C$ nhọn nội tiếp đường tròn $(O)$ với $B, C$ cố định và $A$ di động trên $(O)$. $D$ là trung điểm $B C$. Trên $A B$ lấy các điểm $M, P$ và trên $A C$ lấy các điểm $N, Q$ sao cho $D A=D P=D Q$, dồng thời $D M \perp A C, D N \perp A B$.

(a) Chứng minh rằng các điểm $M, N, P, Q$ cùng thuộc một đường tròn $(\mathcal{C})$ và $(\mathcal{C})$ luôn đi qua một điểm cố định.

(b) Chứng minh rằng tâm của $(\mathcal{C})$ luôn thuộc một đường tròn cố định.

 

Ngày thi thứ hai

Bài 5. Cho số thực $a \neq 0$. Tìm giới hạn của dãy số $\left(u_n\right)$ thoả mãn:

$\quad\quad\quad\quad\quad\quad\quad\quad u_1=0, u_{n+1}\left(u_n+a\right)=a+1, \forall n \in \mathbb{N}^*$

Bài 6. Tìm tất cả các hàm số $f: \mathbb{R}^{+} \rightarrow \mathbb{R}^{+}$thoả mãn điều kiện:

$\quad\quad\quad\quad f\left(x f\left(y^2\right)-y f\left(x^2\right)\right)=(y-x) f(x y) \forall x, y \in \mathbb{R}^{+}, x>y$

Bài 7. Cho $n=2018.2019$. Gọi $A$ là tập hợp các bộ $\left(a_1, a_2, \ldots, a_n\right)$ có thứ tự thoả mãn điều kiện $a_i \in{0,1} \forall i \in{1,2, \ldots, n}$ và $\sum_{i=1}^n a_i=2018^2$. Có bao nhiêu bộ $\left(a_1, a_2, \ldots, a_n\right)$ từ $A$ để $\sum_{i=1}^k a_i \geq \frac{a}{2}$ và $\sum_{i=n-k+1}^n a_i \geq \frac{k}{2} \forall k \in{1,2, \ldots, n}$ ?

Bài 8. Đường tròn $(\mathcal{C})$ tâm $I$ nội tiếp tam giác $A B C$ và tiếp xúc với các cạnh $A B, A C$ tại $E, F$. $A M, A N$ là các đường phân giác trong, phân giác ngoài của góc $\angle B A C(M, N$ nằm trên $B C)$. Gọi $d_M, d_N$ lần lượt là các tiếp tuyến của $(\mathcal{C})$ qua $M, N$ và khác $B C$.

(a) Chứng minh rằng $d_M, d_N, E F$ đồng quy tại điểm $D$.

(b) Lấy trên $A B, A C$ các điểm $P, Q$ thoả mãn $D P|A C, D Q| A B$. Gọi $R, S$ là trung điểm của $D E, D F$. Chứng minh rằng $I$ thuộc đường thẳng qua các trực tâm của hai tam giác $D P S, D Q R$.

 

LỜI GIẢI

Ngày thi thứ nhất

Bài 1. Cho số nguyên $a>1$. Tìm giá trị lớn nhất của số thực $d$ sao cho tồn tại một cấp số cộng có công sai $d$, số hạng đầu tiên là $a$ và có đúng hai trong các số $a^2, a^3, a^4, a^5$ là những số hạng của cấp số cônng đó.

Lời giải: Trước hết, ta chứng minh rằng $d=a^3-a$ thoả mãn điều kiện. Thật vậy, xét cấp số cộng có số hạng đầu là $a$ và công sai là $d=a^3-a$ thì

$\quad\quad\quad\quad\quad\quad\quad\quad\quad\quad \left\{\begin{array}{l}a^3=a+\left(a^3-a\right) \\ a^5=a+\left(a^3-a\right)\left(a^2+1\right)\end{array} .\right.$

Do đó $a^3, a^5$ cùng thuộc cấp số cộng có công sai $d=a^3-a$.

Giả sử rằng tồn tại giá trị $d>a^3-a$ thoả mãn điều kiện bài toán. Khi đó:

$\quad\quad\quad\quad\quad\quad\quad\quad\quad\quad a+d>a+a^3-a=a^3$

Dẫn đến hai số hạng thuộc cấp số cộng phải là $a^4$ và $a^5$. Lại để ý rằng $a>1$ nên có $a<a^4<a^5$, kết hợp lại thì phải tồn tại hai số nguyên dương $k<l$ sao cho:

$\quad\quad\quad\quad\quad\quad\quad\quad\quad\quad \left\{\begin{array}{l}a^4=a+k d \\ a^5=a+l d\end{array}\right.$

Từ đó $a(a+k d)=a+l d$ hay $d(l-a k)=a^2-a>0$.

Chú ý rằng ta có $d>0$ nên $l-a k>0$, hơn nữa $l-a k \in \mathbb{Z}$ nên $l-a k \geq 1$. Điều này dẫn đến $a^2-a \geq d>a^3-a$, vô lý do $a>1$.

Vậy giá trị lớn nhất của $d$ là $\max d=a^3-a$.

Bài 2. Cho $n$ số thực $x_1, x_2, \ldots, x_n$. Với mỗi $i \in{1,2, \ldots, n}$, gọi $a_i$ là số các chỉ số $j$ mà $\left|x_i-x_j\right| \leq 1$ và $b_i$ là số các chỉ số $j$ mà $\left|x_i-x_j\right| \leq 2(i$ và $j$ có thể bằng nhau).

(a) Chứng minh rằng tồn tại $i$ dể $b_i \leq 3 a_i$.

(b) Gọi $A$ là số cặp $(i, j)$ có thứ tự mà $\left|x_i-x_j\right| \leq 1$ và $B$ là số cặp $(i, j)$ có thứ tự mà $\left|x_i-x_j\right| \leq 2$ ( $i$ và $j$ có thể bằng nhau). Chứng minh rằng $B \leq 3 A$.

Lời giải . (a) Không mất tính tổng quát, giả sử $x_1 \leq x_2 \leq \ldots \leq x_n$.

Xét $k=\max [a_1, a_2, \ldots, a_n]$ và $a_i=k$, khi đó tồn tại $k$ số trong dãy là:

$\quad\quad\quad\quad x_u \leq x_{u+1} \leq \ldots \leq x_i \leq \ldots \leq x_v \text { với }\left|x_u-x_i\right|,\left|x_v-x_i\right| \leq 1 .$

Ngoài ra vì tính lớn nhất của $k$ nên $\left|x_{u-1}-x_i\right|>1,\left|x_{v+1}-x_i\right|>1$.

Trong $\left[x_u, x_v\right]$, có đúng $k$ chỉ số $j$ để $\left|x_j-x_i\right| \leq 1<2$. Còn trước $x_u$, xét hai số $x_r, x_s$ sao cho $x_r \leq x_s$ và $\left|x_r-x_i\right| \leq 2,\left|x_s-x_i\right| \leq 2$ thì:

$\quad\quad\quad\quad \left|x_r-x_s\right|=x_s-x_r=\left(x_i-x_r\right)-\left(x_i-x_s\right)<2-1=1$

nên sẽ có không quá $k$ số $j$ để $\left|x_j-x_i\right| \leq 2$ vì nếu ngược lại, sẽ có nhiều hơn $k$ số liên tiếp trong dãy cách nhau không quá 1 đơn vị, mâu thuẫn với tính lớn nhất của $k$. Tương tự với các số sau $x_v$, vì thế nên $b_i \leq 3 k$ kéo theo $b_i \leq 3 a_i$.

(b) Ta sẽ chứng minh bằng quy nạp theo $n$.

Với $n=1$ rõ ràng $A=B=1$ nên khẳng định hiển nhiên đúng. Giả sử kết quả đúng với $n \geq 1$, ta sẽ chứng minh nó cũng đúng với $n+1$.

Xét dãy số thực $T=\left(x_1, x_2, \ldots, x_{n+1}\right)$ bất kỳ và giả sử $x_1 \leq x_2 \leq \ldots \leq x_{n+1}$. Ký hiệu $A_T, B_T$ là số cặp có thứ tự các chỉ số $(i, j)$ tương ứng với định nghĩa của đề bài. Giả sử $k \geq 1$ là số lượng lớn nhất các số của $T$ được chứa trong một đoạn độ dài bằng 2 nào đó.

Gọi $x_i$ là số cuối cùng của dãy mà trong đoạn $\left[x_i-1, x_i+1\right]$ có chứa đúng $k$ số (kể cả $x_i$ ). Gọi $T^{\prime}$ là dãy mới sau khi bỏ $x_i$ đi. Khi đó, số lượng các số thuộc $T^{\prime}$ có trong $\left[x_i-1, x_i+1\right]$ là $k-1$, ngoài ra $x_i$ đã bị bỏ đi thuộc về đúng $2 k-1$ cặp của $A_T$.

Do đó: $A_T=A_{T^{\prime}}+2 k-1$.

Ta viết lại như sau

$\quad\quad\quad\quad \left[x_i-2 ; x_i+2\right]=\left[x_i-2 ; x_i-1\right] \cup\left[x_i-1 ; x_i+1\right] \cup\left[x_i+1 ; x_i+2\right]$

Trừ đoạn ở giữa thì hai đoạn đầu và cuối chứa tối đa $k$ phần tử của $T$. Hơn nữa, do định nghĩa số $x_i$ nên trong đoạn $\left[x_i+1 ; x_i+2\right]$ có tối đa $k-1$ phần tử của $T$. Từ đó có tối đa:

$\quad\quad\quad\quad\quad\quad\quad\quad\quad\quad\quad\quad 2(k-1)+k=3 k-2$

phần tử của $T$ (không tính $x_i$ ) thuộc $\left[x_i-2 ; x_i+2\right]$. Dẫn đến:

$\quad\quad\quad\quad\quad\quad B_T \leq 2(3 k-2)+1+B_{T^{\prime}}=3(2 k-1)+B_{T^{\prime}}$

Áp dụng giả thiết quy nạp, ta có $B_{T^{\prime}}<3 A_{T^{\prime}}$ nên từ các điều trên thì:

$\quad\quad\quad\quad B_T \leq 3(2 k-1)+B_{T^{\prime}}<3(2 k-1)+3 A_{T^{\prime}}=3\left(A_{T^{\prime}}+2 k-1\right)=3 A_T .$

Theo nguyên lý quy nạp, bài toán cũng đúng với $n+1$.

Vậy bài toán được chứng minh hoàn toàn.

Nhận xét. Bài toán này thật ra liên quan đến phương pháp xác suất trong tổ hợp, có thể xem tại quyển “The Probabilistic Method” của GS. Noga Alon. Ta xét một lời giải khác như sau:

(a) Chọn $i$ sao cho số các chỉ số $j$ để $\left|x_i-x_j\right| \leq 1$ là lớn nhất. Khi đó, số lượng chỉ số $j$ sao cho $x_j \in\left(x_i+1, x_i+2\right]$ tối đa là $a_i$, vì nếu không thì tồn tại $j$ để $a_j>a_i$. Tương tự, số lượng chỉ số $j$ sao cho $x_j \in\left[x_i-2, x_i-1\right)$ tối đa là $a_i$.

Chú ý rằng với các chỉ số $j$ để $\left|x_i-x_j\right| \leq 2$ thì ta có điều sau:

$\quad\quad\quad\quad x_j \in\left[x_i-2, x_i-1\right) \cup\left(x_i-1, x_i+1\right) \cup\left(x_i+1, x_i+2\right]$

Số lượng các chỉ số đó chính là $b_i$, dẫn đến $b_i \leq a_i+a_i+a_i=3 a_i$. Hơn nữa, nếu đẳng thức xảy ra, ta phải có mỗi đoạn (hay nửa khoảng) ở phân hoạch trên chứa chính xác $a_i$ chỉ số $j$ của $x_j$.

(b) Bài toán hiển nhiên đúng với $n=1$. Giả sử rằng tồn tại $n>1$ để kết luận không đúng, ta chọn $n$ nhỏ nhất. Ta cũng chọn $i$ sao cho $a_i$ lớn nhất.

Gọi $A^{\prime}, B^{\prime}$ tương ứng là số cặp chỉ số $(k, l)$ mà $\left|x_k-x_l\right| \leq 1$ và $\left|x_k-x_l\right| \leq 2$, trong đó $1 \leq k, l \leq n$ và $k, l \neq i$. Vì $n$ là phản ví dụ nhỏ nhất nên $B^{\prime} \leq 3 A^{\prime}$.

Các cặp chỉ số $(k, l)$ mà $k=i$ hoặc $l=i$ và $\left|x_k-x_l\right| \leq 1$ đều phải có dạng $(k, i)$ hoặc $(i, k)$ trong đó $k \neq i$ và $(i, i)$. Có tổng cộng $2\left(a_i-1\right)+1$ cặp như thế nên $A=A^{\prime}+2\left(a_i-1\right)+1$.

Tương tự thì $B=B^{\prime}+2\left(b_i-1\right)+1$. Do đó nếu $b_i \leq 3 a_i-1$ thì:

$\quad\quad\quad\quad B=B^{\prime}+2 b_i-1 \leq 3 A^{\prime}+2\left(3 a_i-1\right)-1=3\left(A^{\prime}+2 a_i-1\right)=3 A$

Điều này trái với việc $n$ là phản ví dụ nhỏ nhất. Do đó $b_i \geq 3 a_i$. Theo ý (a) thì $b_i \leq 3 a_i$, từ đây phải có $b_i=3 a_i$. Hơn nữa, số lượng chỉ số $j$ để thỏa mãn $x_j \in\left[x_i-2, x_i-1\right)$ hoặc $x_j \in\left(x_i+1, x_i+2\right]$ dều phải bằng $a_i$.

Với mỗi $j, j^{\prime}$ sao cho $x_j, x_{j^{\prime}} \in\left[x_i-2, x_i-1\right)$, ta có $\left|x_j-x_{j^{\prime}}\right|<1$, dẫn đến $a_j \geq a_i$. Mặt khác $a_i$ là lớn nhất có thể nên $a_j=a_i$. Tương tự, với mỗi $j$ sao cho $x_j \in\left(x_i+1, x_i+2\right]$ thì $a_j=a_i$. Như vậy với mọi $j$ sao cho $1<\left|x_i-x_j\right| \leq 2$ thì $a_j=a_i$. Cũng với cách chọn chỉ số $j$ đó, lập luận tương tự như những ý trên, ta cũng phải có $b_j=3 a_j$.

Xây dựng đồ thị $\mathcal{G}$ với các đỉnh được đánh số là $1,2, \ldots, n$ sao cho cặp đỉnh $(k, l)$ kề nhau khi và chỉ khi $1<\left|x_k-x_l\right| \leq 2$. Những lập luận trên cho thấy mọi đỉnh $j$ mà tồn tại một đường đi từ $i$ đến $j$ đều phải thỏa mãn $a_j=a_i$ và $b_j=3 a_j$. Gọi $\mathcal{X}$ là tập hợp tất cả các đỉnh $j$ sao cho tồn tại một đường đi từ $i$ dến $j$ trong $\mathcal{G}$. Đặt $\mathcal{Y}={1,2, \ldots, n} \backslash \mathcal{X}(\mathcal{Y}$ có thể rỗng $)$.

Bây giờ, gọi $A_y, B_y$ tương ứng là số cặp chỉ số $(k, l)$ có tính thứ tự, có thể bằng nhau mà $\left|x_k-x_l\right| \leq 1$ và $\left|x_k-x_l\right| \leq 2$, trong đó $k, l \in \mathcal{Y}$. Chú ý rằng $A_{\mathcal{Y}}=B_{\mathcal{Y}}=0$ nếu $\mathcal{Y}=\emptyset$. Bởi $n$ là phản ví dụ nhỏ nhất, ta phải có $B_{\mathcal{Y}} \leq 3 A_{\mathcal{Y}}$. Ta gọi $a_{y, k}$ và $b_{y, k}$ tương ứng là số chỉ số $j \in \mathcal{Y}$ mà $\left|x_j-x_k\right| \leq 1$ và $\left|x_j-x_k\right| \leq 2$. Định nghĩa tương tự $a_{\mathcal{X}, k}$ và $b_{\mathcal{X}, k}$.

Với mọi $k \in \mathcal{Y}$, dễ thấy $k$ không kề bất cứ đỉnh nào trong $\mathcal{X}$, vì vậy ta có được $b_{\mathcal{X}, k}=0$ và $b_k=b_{\mathcal{Y}, k}+a_{\mathcal{X}, k}$. Từ đây dẫn đến đẳng thức sau:

$\quad\quad\quad\quad\quad\quad B=\sum_{k \in \mathcal{X}} b_k+\sum_{k \in \mathcal{Y}} b_k=3 \sum_{k \in \mathcal{X}} a_k+\sum_{k \in \mathcal{Y}}\left(b_{y, k}+a_{\mathcal{X}, k}\right)$

Ta đồng thời có $\sum_{k \in \mathcal{Y}} b_{y, k}=B_{\mathcal{Y}} \leq 3 A_{\mathcal{Y}}$. Hơn nữa, ta cũng có được:

$\quad\quad A=\sum_{k \in \mathcal{X}} a_k+\sum_{k \in \mathcal{Y}} a_k=\sum_{k \in \mathcal{X}} a_k+\sum_{k \in \mathcal{Y}}\left(a_{\mathcal{Y}, k}+a_{\mathcal{X}, k}\right)=\sum_{k \in \mathbb{X}} a_k+A_{\mathcal{Y}}+\sum_{k \in \mathcal{Y}} a_{\mathcal{X}, k}$

Do đó:

$\quad\quad\quad\quad B \leq 3 A_{\mathcal{Y}}+\sum_{k \in \mathcal{Y}} a_{\mathcal{X}, k}+3 \sum_{k \in \mathcal{X}} a_k \leq 3\left(A_{\mathcal{Y}}+\sum_{k \in \mathcal{Y}} a_{\mathcal{X}, k}+\sum_{k \in \mathcal{X}} a_k\right)=3 A$

Điều này dẫn đến giả sử phản chứng là sai.

Vì vậy, với mọi số nguyên dương $n$, ta phải có $B \leq 3 A$. Bài toán kết thúc.

Bài 3. Cho $p$ là số tự nhiên. Xét phương trình nghiệm nguyên

$\quad\quad\quad\quad\quad\quad\quad\quad\quad\quad\quad\quad x^3+x+p=y^2 .$

(a) Tìm số nguyên tố $p$ nhỏ nhất dạng $4 k+1$ sao cho phương trình có nghiệm.

(b) Chứng minh rằng nếu $p$ là số chính phương thì phương trình trên có nghiệm nguyên dương.

Lời giải. (a) Các số nguyên tố có dạng $4 k+1$ là $5,13,17, \ldots$

Trước hết, ta thấy với $p=13$ thì $x^3+x+13=y^2$ có nghiệm là $(x ; y)=(4 ; 9)$. Ta sẽ chứng minh rằng phương trình $x^3+x+5=y^2$ không có nghiệm nguyên. Xét modulo 4. Có các khả năng sau xảy ra:

  • Khi $x$ chia 4 dư $0,1,2,3$, vế trái chia 4 lần lượt dư $1,3,3,3$.
  • Khi $y$ chia 4 dư $0,1,2,3$, vế phải chia 4 lần lượt dư $0,1,0,1$.

Do đó $y$ phải lẻ và $4 \mid x$. Viết biểu thức đã cho thành:

$\quad\quad\quad\quad\quad\quad\quad\quad\quad\quad (x+3)\left(x^2-3 x+10\right)=y^2+5^2$

Do $x+3 \equiv 3(\bmod 4)$ nên $x+3$ có ước nguyên tố $q \equiv 3(\bmod 4)$. Ta biết rằng với $a, b \in \mathbb{Z}$ thì $a^2+b^2$ chia hết cho số nguyên tố $q \equiv 3(\bmod 4)$ khi và chỉ khi $q \mid a$ và $q \mid b$. Từ đó thì $q \mid 5$ hay $q=5$, mâu thuẫn.

Vậy $p=13$ là số nguyên tố nhỏ nhất cần tìm.

(b) Trước hết, ta giới thiệu kết quả sau (còn gọi là định lý 4 số):

Bổ Đề. Với các số nguyên dương $a, b, c, d$ thoả mãn $a b=c d$ thì tồn tại các số nguyên dương $x, y, z, t$ sao cho $a=x y, b=z t, c=x z, d=y t$.

Chứng minh bổ đề. Đặt $k=\operatorname{gcd}(a, c)$ và viết $a=k a_1, c=k c_1$ thì rõ ràng $\operatorname{gcd}\left(a_1, c_1\right)=1$. Thay vào đề bài, ta có

$\quad\quad\quad\quad\quad\quad\quad\quad\quad\quad k a_1 b=k c_1 d \text { hay } a_1 b=c_1 d .$

Từ đây chú ý $a_1 \mid c_1 d$, nên $a_1 \mid d$, đặt $d=a_1 \ell$. Thay vào thì có $b=\ell c_1$. Từ đó, ta chọn $x=k, y=a_1, z=c_1, t=\ell$ thì có ngay điều phải chứng minh.

Quay lại bài toán, do $p$ là số chính phương nên đặt $p=a^2, a \in \mathbb{Z}$. Ta viết lại phương trình thành dạng:

$\quad\quad\quad\quad\quad\quad\quad\quad x^3+x+a^2=y^2 \text { hay } x\left(x^2+1\right)=(y-a)(y+a) .$

Áp dụng kết quả trên vào bài toán, ta thấy tồn tại các số nguyên dương $m, n, p, q$ để $x=m n, x^2+1=p q, y+a=m p, y-a=n q$. Từ đó:

$\quad\quad\quad\quad\quad\quad\quad\quad\quad\quad (m n)^2+1=p q \text { và } m p-n q=2 a \text {. }$

Xét dãy số $\left(u_n\right)$ xác định bởi $u_0=0, u_1=1, u_{n+2}=\alpha u_{n+1}+u_n$, trong đó $\alpha$ là hằng số mà ta sẽ chọn sau. Rõ ràng với mọi $n$ thì

$\quad\quad\quad\quad\quad\quad\quad\quad u_n^2-u_{n+1} u_{n-1}=(-1)^{n-1}\left(u_1^2-u_2 u_0\right)=(-1)^{n-1} .$

Khi đó, với $n$ chẵn thì $u_n^2-u_{n+1} u_{n-1}=-1$. Chọn $m n=u_{2 k}$. Ta có:

$\quad\quad\quad\quad\quad\quad\quad u_2=\alpha, u_3=\alpha^2+1, u_4=\alpha\left(\alpha^2+2\right), u_5=\alpha^4+3 \alpha^2+1$

Chọn $p=u_3, q=u_5, m n=u_4$ thì rõ ràng $(m n)^2+1=p q$. Bây giờ ta chỉ cần có được

$\quad\quad\quad\quad\quad\quad m u_3-n u_5=2 a \text { hay } m\left(\alpha^2+1\right)-n\left(\alpha^4+3 \alpha^2+1\right)=2 a .$

Từ đây chọn $\alpha=4 a^2$ và viết $m=2 a\left(\alpha^2+2\right), n=2 a\left(\alpha^4+3 \alpha^2+1\right)$ thì đẳng thức trên sẽ thoả mãn, vì

$\quad\quad\quad\quad\quad\quad\quad\quad \left(\alpha^2+1\right)\left(\alpha^2+2\right)-\left(\alpha^4+3 \alpha^2+1\right)=1 .$

Vậy phương trình có một cặp nghiệm cụ thể là

$\quad\quad (x, y)=\left(4 a^2\left(16 a^4+2\right), 2 a\left(16 a^4+2\right)\left(16 a^4+1\right)-a\right) \text { với } a=\sqrt{p} \in \mathbb{Z}^{+} .$

Bài 4. Cho tam giác $A B C$ nhọn nội tiếp đường tròn $(O)$ với $B, C$ cố định và $A$ di động trên $(O)$. $D$ là trung điểm $B C$. Trên $A B$ lấy các điểm $M, P$ và trên $A C$ lấy các điểm $N, Q$ sao cho $D A=D P=D Q$, dồng thời $D M \perp A C, D N \perp A B$.

(a) Chứng minh rằng các điểm $M, N, P, Q$ cùng thuộc một đường tròn $(\mathcal{C})$ và (C) luôn đi qua một điểm cố định.

(b) Chứng minh rằng tâm của $(\mathcal{C})$ luôn thuộc một đường tròn cố định.

Lời giải . (a) Dễ thấy tam giác $A M Q$ cân tại $M$ nên

$\quad\quad \angle D M Q=\angle D M A=90^{\circ}-\angle A=\frac{180^{\circ}-2 \angle A}{2}=\frac{180^{\circ}-\angle P D Q}{2}=\angle D P Q$

Do đó tứ giác $M P D Q$ nội tiếp. Chứng minh tương tự, ta có tứ giác $Q N D P$ nội tiếp nên $M, N, P, Q$ cùng thuộc một đường tròn $(\mathcal{C})$, và $(\mathcal{C})$ luôn đi qua điểm $D$ cố định.

(b) Gọi $K B, K C$ là hai tiếp tuyến của $(O)$. Ta có $D, K, O$ thẳng hàng, lại có:

$\quad\quad\quad\quad\quad \angle B K O=90^{\circ}-\angle B O K=90^{\circ}-\angle B A C=\angle B M D$

Từ đó tứ giác $B D K M$ nội tiếp. Để ý rằng $K D \perp B C$ nên $K M \perp A B$, hơn nữa $D N \perp A B$ nên $K M | D N$. Tương tự thì $K N | D M$. Do đó $D M K N$ là hình bình hành hay $D K, M N$ có $J$ là trung điểm chung.

Gọi $I$ là tâm của $(\mathcal{C})$ thì $I J \perp M N$ và $J L | A D$. Chú ý rằng $D$ là tâm $(A P Q)$ và cũng là trực tâm tam giác $A M N$ nên $P Q, M N$ là hai đường đối song. Đồng thời nếu $L$ là trung điểm $A D$ thì $J L$ vuông góc với đường nối hai chân đường cao từ $M, N$ của tam giác $A M N$ nên $J L \perp P Q$. Lại có $D P=D Q$ và $I P=I Q$ nên $I D \perp P Q$, do đó $J L | D I$.

Từ đây $I D L J$ là hình bình hành và $I L, D J$ có $T$ là trung điểm chung cố định. Xét phép vị tự tâm $D$ tỉ số $\frac{1}{2}$ hợp với phép đối xứng tâm $T$ thì $A \mapsto I$. Do $A$ thuộc đường tròn $(O)$ cố định nên $I$ cũng thuộc đường tròn cố định là ảnh của $(O)$ qua hợp các phép biến hình trên. Bài toán kết thúc.

Nhận xét. Bài toán này còn một hướng tiếp cận bản chất hơn như sau. Nếu gọi $A^{\prime}$ là điểm đối xứng của $A$ qua $D$ thì $K, A^{\prime}$ là hai điểm liên hợp đẳng giác trong tam giác $A B C$, từ đó đường tròn $(\mathcal{C})$ chính là đường tròn đi qua các hình chiếu của $K, A^{\prime}$ trên các cạnh tam giác $A B C$, dồng thời $I$ là trung diểm $K A^{\prime}$.

Dưới đây là một bài toán tương tự: Cho tam giác nhọn $A B C$ nội tiếp đường tròn $(O)$ có $B C$ cố định và $A$ di dộng trên $(O)$. Gọi $H$ là trực tâm tam giác và lấy điểm $E, F$ thuộc $A B, A C$ theo thứ tự đó sao cho $H$ là trung điểm $E F$.

  1. Chứng minh rằng tâm của đường tròn $(A E F)$ luôn thuộc một đường tròn cố định. Đặt là $\omega$.
  2. Giả sử $\omega$ cắt lại $(O)$ tại các điểm $X, Y$. Chứng minh rằng $X, Y, O$ thẳng hàng.

 

Ngày thi thứ hai

Bài 5. Cho số thực $a \neq 0$. Dãy số $\left(u_n\right)$ thoả mãn:

$\quad\quad\quad\quad\quad\quad\quad\quad\quad\quad u_1=0, u_{n+1}\left(u_n+a\right)=a+1 \forall n \in \mathbb{N}^*$

Tìm giới hạn của dãy số $\left(u_n\right)$.

Lời giải: Đặt $x_{n+1}=(a+1) y_n$ và $y_{n+1}=x_n+a y_n$. Ta có:

$\quad\quad\quad\quad\quad\quad\quad y_{n+2}=x_{n+1}+a y_{n+1}=a y_{n+1}+(a+1) y_n$

Đồng thời $u_n=\frac{x_n}{y_n}$. Để ý rằng $u_1=0, u_2=\frac{a+1}{a}$. Chọn $y_1=1, y_2=a$. Từ đó:

$\quad\quad\quad\quad\quad\quad\quad\quad\quad\quad y_n=\frac{(a+1)^n-(-1)^n}{a+2} \forall n \geq 1$

Công thức trên chỉ xác định với $a \neq-2$ nên xét trường hợp $a=-2$, ta có dãy

$\quad\quad\quad\quad\quad\quad\quad\quad\quad\quad \left\{\begin{array}{l}u_1=0, \\ u_{n+1}=\frac{1}{2-u_n}, n \geq 1\end{array} .\right.$

Bằng quy nạp, ta chứng minh được $u_n \in[0 ; 1)$ nên:

$\quad\quad\quad\quad\quad\quad\quad u_{n+1}-u_n=\frac{1}{2-u_n}-u_n=\frac{\left(u_n-1\right)^2}{2-u_n}>0$

Dãy $\left(u_n\right)$ tăng và bị chặn trên bởi 1 nên có giới hạn hữu hạn là $L \in(0,1)$. Giải phương trình giới hạn, ta có được $L=\frac{1}{2-L}$. Khi đó thì $L=1$.

Tiếp theo, xét $a \neq-2$, ta có:

$\quad\quad\quad\quad u_n=\frac{x_n}{y_n}=\frac{(a+1) y_{n-1}}{y_n}=\frac{(a+1)^n+(a+1)(-1)^n}{(a+1)^n-(-1)^n} \forall n \in \mathbb{N}^*$

Đặt $-(a+1)=b \in{-1 ; 1}$, ta viết lại thành:

$\quad\quad\quad\quad\quad\quad\quad\quad\quad\quad u_n=\frac{b^n-b}{b^n-1} \forall n \geq 1$

Có các khả năng sau xảy ra:

  • Nếu $b>1$ hoặc $b<-1$, tương ứng là $a<-2$ hoặc $a>0$, thì $\lim u_n=1$.
  • Nếu $-1<b<1$, tương ứng là $-2<a<0$, thì $\lim u_n=b=-(a+1)$.

Vậy ta có kết luận sau trong các trường hợp của $a$ :

  • Nếu $a \in(-2 ; 0)$ thì $\lim u_n=-(a+1)$.
  • Nếu $a \notin(-2 ; 0)$ thì $\lim u_n=-1$.

Bài 6. Tìm tất cả các hàm số $f: \mathbb{R}^{+} \rightarrow \mathbb{R}^{+}$thoả mãn diều kiện:

$\quad\quad\quad\quad\quad\quad f\left(x f\left(y^2\right)-y f\left(x^2\right)\right)=(y-x) f(x y) \forall x, y \in \mathbb{R}^{+}, x<y .$

Lời giải . Theo giả thiết thì với mọi $y>x>0$, ta đều có

$\quad\quad\quad\quad\quad\quad\quad\quad x f\left(y^2\right)-y f\left(x^2\right)>0 \Rightarrow \frac{f\left(y^2\right)}{f\left(x^2\right)}>\frac{y}{x}>1 .$

Do đó,

$\quad\quad\quad\quad\quad\quad\quad\quad y^2>x^2 \Leftrightarrow y>x \Leftrightarrow f\left(y^2\right)>f\left(x^2\right)$

nên hàm $f$ dã cho đồng biến trên $\mathbb{R}^{+}$. Trong đề bài, thay $y=x+1$, ta có

$\quad\quad\quad\quad\quad\quad\quad f\left(x f\left((x+1)^2\right)-(x+1) f\left(x^2\right)\right)=f(x(x+1))$

hay

$\quad\quad\quad\quad\quad\quad\quad\quad x f\left((x+1)^2\right)-(x+1) f\left(x^2\right)=x(x+1) $

$\quad\quad\quad\quad\quad\quad\quad\quad \Leftrightarrow \frac{f\left((x+1)^2\right)}{x+1}=\frac{f\left(x^2\right)}{x}+1, \forall x>0$

Thực hiện thao tác này nhiều lần, ta có

$\quad\quad\quad\quad\quad\quad\quad\quad \frac{f\left((x+n)^2\right)}{x+n}=\frac{f\left(x^2\right)}{x}+n, \forall x>0, n \in \mathbb{Z}^{+}$

hay

$\quad\quad\quad\quad\quad\quad\quad\quad x f\left((x+n)^2\right)-(x+n) f\left(x^2\right)=n x(x+n) .$

Trong dề bài, thay $y=x+n$, ta có

$\quad\quad\quad\quad\quad\quad f\left(x f\left((x+n)^2\right)-(x+n) f\left(x^2\right)\right)=n f(x(x+n)) $

$\quad\quad\quad\quad\quad\quad \Leftrightarrow f(n x(x+n))=n f(x(x+n)) .$

Với mọi $n \in \mathbb{Z}^{+}, y>0$, ta luôn chọn được $x>0$ để $x(x+n)=y$ nên ta có

$\quad\quad\quad\quad\quad\quad\quad\quad\quad\quad f(n y)=n f(y), \forall n \in \mathbb{Z}^{+}, y \in \mathbb{R}^{+} .$

Đặt $f(1)=a>0$, với mọi $n \in \mathbb{Z}^{+}$, cho $y=\frac{1}{n}$, suy ra

$\quad\quad\quad\quad\quad\quad\quad\quad\quad\quad f(1)=n f\left(\frac{1}{n}\right) \Rightarrow f\left(\frac{1}{n}\right)=\frac{a}{n} .$

Do đó,

$\quad\quad\quad\quad\quad\quad\quad\quad f\left(\frac{n}{m}\right)=n f\left(\frac{1}{m}\right)=\frac{n}{m} a, \forall m, n \in \mathbb{Z}^{+}$

hay $f(x)=a x, \forall x \in \mathbb{Q}^{+}$. Với mọi số thực $x_0>0$, chọn hai dãy số hữu tỷ $\left(a_n\right),\left(b_n\right)$ sao cho $a_n<x_0<b_n$ và $\lim a_n=\lim b_n=x_0$. Rõ ràng

$\quad\quad\quad\quad\quad\quad f\left(a_n\right)<f\left(x_0\right)<f\left(b_n\right) \Rightarrow a \cdot a_n<f\left(x_0\right)<a \cdot b_n,$

nên cho $n \rightarrow+\infty$, ta có $f\left(x_0\right)=a x_0$. Do đó, với mọi số thực $x>0$ thì $f(x)=a x$. Thay vào biểu thức đã cho, ta có

$\quad\quad\quad\quad\quad \left\{\begin{array}{l}f\left(x f\left(y^2\right)-y f\left(x^2\right)\right)=a^2\left(x y^2-x^2 y\right)=a^2(y-x) x y \\ (y-x) f(x y)=a(y-x) x y\end{array}\right.$

nên $a=1$. Vậy tất cả các hàm số cần tìm là $f(x)=x, \forall x>0$.

Nhận xét. Có một điều đáng chú ý ở bài toán này là việc từ giả thiết, ta phải ngầm hiểu rằng $x f\left(y^2\right)-y f\left(x^2\right)>0$ với mọi cặp số dương $x<y$. Ta có thể thêm tường minh điều kiện đó vào đề bài cho rõ. Tuy nhiên, nếu thêm theo kiểu như sau thì sẽ có một chút vấn đề phát sinh:

Tìm tất cả các hàm số $f: \mathbb{R}^{+} \rightarrow \mathbb{R}^{+}$thoả mãn với mọi cặp số dương $x<y$, nếu $x f\left(y^2\right)-y f\left(x^2\right)>0$ thì

$\quad\quad\quad\quad\quad\quad\quad\quad\quad\quad f()=(y-x) f(x y) \forall x, y \in \mathbb{R}^{+}, x<y .$

Khi đó, ta có thể nhận thêm một hàm số thỏa mãn nữa là $f(x)=\sqrt{x}$. Lý do là vì với mọi cặp số $y>x>0$, ta đều có $x f\left(y^2\right)-y f\left(x^2\right)=0$, mà vì thế, điều kiện “nếu” ở trên là sai nên mệnh đề kéo theo là đúng.

Bài 7. Cho $n=2018.2019$. Gọi $A$ là tập hợp các bộ $\left(a_1, a_2, \ldots, a_n\right)$ có thứ tự thoả mãn điều kiện $a_i \in{0,1} \forall i \in{1,2, \ldots, n}$ và $\sum_{i=1}^n a_i=2018^2$.

Có bao nhiêu bộ $\left(a_1, a_2, \ldots, a_n\right)$ từ $A$ dể:

$\quad\quad\quad\quad\quad \sum_{i=1}^k a_i \geq \frac{a}{2} \text { và } \sum_{i=n-k+1}^n a_i \geq \frac{k}{2} \forall k \in{1,2, \ldots, n}$

Lời giải. Ta giải bài toán tổng quát khi thay 2018 bởi $m \in \mathbb{Z}^{+}$. Bài toán đã cho tương đương với bài toán sau:

Trong hệ trục tọa độ Oxy, xét lưới điểm nguyên trong hình chũ nhật có đỉnh dưới bên trái là $O(0 ; 0)$ và dỉnh trên bên phải là $A\left(m^2 ; m\right)$. Dặt $B(m ; m)$ và $C\left(m^2-m ; 0\right)$, hỏi có bao nhiêu đương đi tù̀ $O \rightarrow A$ sao cho mỗi bước, ta đi sang phải hoặc lên trên 1 đơn vị, gọi là đương đi đơn, và không vượt lên trên $O B$ cũng nhu không xuống dưới $A C$ ?

Ở đây, các số $0 ; 1$ tương ứng với các bước đi lên trên, các bước đi sang phải; còn điều kiện tổng $k$ số đầu và tổng $k$ số cuối không nhỏ hơn $\frac{k}{2}$ tương ứng với số lượng bước đi lên không vượt quá số lượng bước đi sang phải. Để thuận tiện, ta gọi đường đi cắt $d$ nếu nó có các phần nằm về cả hai phía của $d$. Trước hết, ta sẽ chứng minh bổ đề sau:

Bổ Đề. Số đường đi đơn từ $O \rightarrow A(m ; n)$, có cắt đường thẳng $y=x$, là $C_{m+n}^{m+1}$.

Thật vậy, Xét đường thẳng $(d): y=x+1$, rõ ràng các đường đi đơn cắt $y=x$ dều sẽ có điểm chung với đường thẳng $(d)$ này. Tại các điểm chung đó, ta thực hiện đối xứng trục để được một đường đi mới xuất phát từ $O \rightarrow A^{\prime}(n-1, m+1)$.

Trong hình trên, đường cũ là đứt nét, còn đường mới là liền nét. Rõ ràng phép đối xứng trục trên là song ánh, biến các đường cần tìm (cắt $y=x$ ), thành các đường từ $O \rightarrow A^{\prime}$; do đó, số lượng đường cần tìm là $C_{m+n}^{n-1}$.

Trở lại bài toán,

Số đường đi đơn từ $O \rightarrow A\left(m^2 ; m\right)$ là $C_{m^2+m}^m$ vì nó bằng số cách chọn $m$ lần đi lên trong tổng số $m^2+m$ lần di chuyển, trong đó số đường đi cắt $O B$ bằng số đường đi cắt $A C$ và bằng $C_{m^2+m}^{m-1}$ (theo bổ đề).

Do đó, ta chỉ cần tìm số đường đi cắt cả $O B, A C$ với ý tưởng đối xứng hai lần đã dùng để chứng minh bổ đề.

Đầu tiên, ta thực hiện đối xứng qua đường thẳng $y=x+1$; khi đó, các đường đi đơn sẽ xuất phát từ $O \rightarrow A^{\prime}\left(m-1 ; m^2+1\right)$. Do các đường ban đầu còn vượt qua $A C$ nên các đường mới phải cắt thêm $y=x+m^2-m+3$. Tiếp tục đối xứng qua đường thẳng này, ta đưa về đếm số đường đi đơn từ $O \rightarrow A^{\prime \prime}\left(m-2, m^2+2\right)$. Suy ra số đường đi trong trường hợp này là $C_{m^2+m}^{m-2}$. Vậy theo nguyên lý bù trừ, kết quả cần tìm sẽ là

$\quad\quad\quad\quad\quad\quad\quad\quad\quad\quad C_{m^2+m}^m-2 C_{m^2+m}^{m-1}+C_{m^2+m}^{m-2} .$

Thay $m=2018$, ta có số lượng đường đi, cũng chính là số bộ thỏa mãn đề bài.

Nhận xét. Dưới đây là một số kết quả tương tự về đường đi đơn trong đề bài

$1$. Số đường đi đơn từ $(0 ; 0) \rightarrow(m ; n)$ mà không có điểm chung với $y=x$ là

$\quad\quad\quad\quad\quad\quad\quad\quad\quad\quad\quad\quad \frac{m-n}{m+n} C_{m+n}^m .$

$2$. Số đường đi đơn từ $(0 ; 0) \rightarrow(m ; n)$ mà không vượt qua $y=x$ là

$\quad\quad\quad\quad\quad\quad\quad\quad\quad\quad\quad C_{m+n}^n-C_{m+n}^{n-1} .$

$3$. Số đường đi gồm $n$ bước mà không vượt $y=x$ là

$\quad\quad\quad\quad\quad\quad\quad\quad\quad \sum_{i=n / 2}^n \frac{n !(2 i+1-n)}{(i+1) !(n-i) !}=C_n^{[n / 2]} .$

$4$. Số đường đi đơn từ $(0 ; 0) \rightarrow(m ; n)$ mà không có điểm chung với $y=x+t$ là

$\quad\quad\quad\quad\quad\quad\quad\quad\quad\quad\quad\quad C_{m+n}^n-C_{m+n}^{m-t}$.

Bạn đọc có thể dùng phương pháp tương tự trên để giải quyết các bài toán này.

Bài 8. Đường tròn $(\mathcal{C})$ tâm $I$ nội tiếp tam giác $A B C$ và tiếp xúc với các cạnh $A B, A C$ tại $E, F$. $A M, A N$ là các đường phân giác trong, phân giác ngoài của góc $\angle B A C(M, N$ nằm trên $B C)$. Gọi $d_M, d_N$ lần lượt là các tiếp tuyến của $(\mathcal{C})$ qua $M, N$ và khác $B C$.

(a) Chứng minh rằng $d_M, d_N, E F$ dồng quy tại điểm $D$.

(b) Lấy trên $A B, A C$ các điểm $P, Q$ thoả mãn $D P|A C, D Q| A B$. Gọi $R, S$ là trung điểm của $D E, D F$. Chứng minh rằng $I$ thuộc đường thẳng qua các trực tâm của hai tam giác $D P S, D Q R$.

Lời giải. (a) Gọi $X, Y$ lần lượt là tiếp điểm của tiếp tuyến thứ hai kẻ từ $M$ dến $(I)$ và $D^{\prime}$ là tiếp điểm của $(I)$ trên $B C$. Gọi $K$ là trung điểm $E F$.

Xét trong đường tròn $(I)$ thì $E F$ là đường đối cực của $A$ và $K \in E F$ nên đối cực của $K$ sẽ đi qua $A$, mà $N A \perp I A$ nên $N A$ chính là đường đối cực của $K$.

Đường đối cực của $K$ đi qua $N$ nên đối cực của $N$, là $D^{\prime} Y$, sẽ đi qua $K$. Dễ thấy rằng $A M$ là trục đối xứng của tứ giác $D^{\prime} X E F$ nên suy ra $D^{\prime} X | E F$. Xét $D^{\prime}(E F, X Y)$, ta có có $D^{\prime} Y$ đi qua trung điểm của $E F$ và $D^{\prime} X | E F$ nên

$\quad\quad\quad\quad\quad\quad\quad\quad\quad\quad D^{\prime}(E F, X Y)=-1$

hay tứ giác $E X F Y$ điều hòa. Suy ra $M X, N Y, E F$ đồng quy. Ngoài ra ta cũng có $X, Y, A$ thẳng hàng.

(b) Dễ thấy các tam giác $P E D$ và $D Q F$ là các tam giác cân. Gọi $H_1, H_2$ lần lượt là trực tâm của tam giác $\triangle D P S, \triangle D Q R$. Ta có

$\quad\quad\quad\quad\quad\quad\quad\quad \angle P H_1 S=\angle P D F=\angle A F E=\angle P E S$

nên $E P S H_1$ là tứ giác nội tiếp. Suy ra $R H_1 \cdot R P=R S \cdot R E$. Ngoài ra,

$\quad\quad\quad\quad\quad\quad\quad\quad\quad\quad K A \cdot K I=K E \cdot K F$

nên

$\quad\quad\quad\quad\quad\quad\quad\quad\quad \frac{R P}{K A} \cdot \frac{R H_1}{K I}=\frac{R E}{K E} \cdot \frac{R S}{K F} .$

Theo định lý Thales thì $\frac{R P}{K A}=\frac{R E}{K E}$ nên $\frac{R H_1}{K I}=\frac{R S}{K F}$, mà

$\quad\quad\quad\quad\quad\quad\quad R S=R D-S D=\frac{D E-D F}{2}=\frac{E F}{2}=K F$

Suy ra $R H_1=K I$, mà $R H_1 | K I$ (do cùng vuông góc với $E F$ ) nên $I K R H_1$ là hình chữ nhật, kéo theo $I H_1 | E F$. Một cách tương tự, ta có $I H_2 | E F$ vậy nên đường thẳng $H_1 H_2$ đi qua $I$.

Nhận xét. Trong câu a, tính chất $A, X, Y$ thẳng hàng của bài toán cũng đúng khi thay $M, N$ là chân các đường phân giác bởi cặp điểm liên hợp điều hòa bất kỳ với $B, C$. Điều này có được nhờ tính chất của các đường đối cực (hoặc có thể chứng minh nhờ việc sử dụng phép chiếu trực giao các chùm điều hòa).